Középiskolai Matematikai és Fizikai Lapok
Informatika rovattal
Kiadja a MATFUND Alapítvány
Már regisztráltál?
Új vendég vagy?

Fórum: Fizikások válaszoljanak

  [1]    [2]    [3]    [4]    [5]    [6]    [7]  

Szeretnél hozzászólni? Jelentkezz be.
[985] szekibarbi2012-05-18 11:30:33

Egy kereszteződésben állunk. A közeledő mentőautó szirénájának hangját 560 Hz-nek érzékeljük amikor közeledik és 465 Hz-nek amikor távolodik. (hang sebessége 343 m/s) Mekkora a mentőautó sebessége (m/s)?

[984] gorgi2012-05-18 11:17:00

f=f0*c/(c+v)

df/dt = f0*c * d(1/(c+v)) / dt = -f0*c/(c+v)2 * dv/dt

(c+v)2: olvasandó: (c+v) négyzet

ahol dv/dt= a megadott gyorsulás (láncszabály)

(maga v pedig ugyanúgy, mint az előbb, a becsapódás előtti pillanatban)

Előzmény: [980] szekibarbi, 2012-05-18 10:22:36
[983] szekibarbi2012-05-18 11:08:52

Egy kereszteződésben állunk. A közeledő mentőautó szirénájának hangját 560 Hz-nek érzékeljük amikor közeledik és 465 Hz-nek amikor távolodik. (hang sebessége 343 m/s) Mekkora a mentőautó sebessége (m/s)?

[982] szekibarbi2012-05-18 10:59:11

nem jön ki...

Előzmény: [981] Gézoo, 2012-05-18 10:56:00
[981] Gézoo2012-05-18 10:56:00

Húha, tényleg! Bocsánatot kérek mindenkitől! Valóban f=f*c/(c+v) a helyes alak.

Köszönöm a javítást!

Előzmény: [978] gorgi, 2012-05-18 10:08:49
[980] szekibarbi2012-05-18 10:22:36

hű, hát nem az igazi...

Előzmény: [979] gorgi, 2012-05-18 10:10:39
[979] gorgi2012-05-18 10:10:39

Hűha, úgy látom, a tördeléssel nem sikerült megbírkóznom. Ha érthetetlen így, szólj, és megpróbálom igazítani :((

Előzmény: [978] gorgi, 2012-05-18 10:08:49
[978] gorgi2012-05-18 10:08:49

Gézoo előbbi megoldása szerintem egy helyen téves. A dopplert tévesen írta fel.

A helyes képlet: f=f0*c/(c+v) (=760 1/s) a frekvenciaváltozáshoz:

f=f0*c/(c+v) df/dt=f0*c * d(1/(c+v)) / dt = -f0*c/(c+v)2 * dv/dt ((c+v) négyzet) ahol dv/dt= a megadott gyorsulás (láncszabály) (maga v pedig ugyanúgy, mint az előbb, a becsapódás előtti pillanatban)

Előzmény: [974] szekibarbi, 2012-05-18 09:13:15
[977] szekibarbi2012-05-18 09:35:29

feltételezem, igen

Előzmény: [975] Gézoo, 2012-05-18 09:34:16
[976] szekibarbi2012-05-18 09:34:49

hát nem tudom :)

Előzmény: [975] Gézoo, 2012-05-18 09:34:16
[975] Gézoo2012-05-18 09:34:16

Nem egészen értem a kérdést. Ahogy én értem, a gyorsulás a=dv/dt függvényét az f=f0*(c-v)/c függvényébe kellene bele "kombinálni" úgy, hogy a sebesség helyére a sebesség változás, azaz a gyorsulás kerülhessen.

A frekvencia gyorsulásának azaz az af=df/dt függvényét kellene képeznünk?

Előzmény: [974] szekibarbi, 2012-05-18 09:13:15
[974] szekibarbi2012-05-18 09:13:15

köszönöm, a feladat hasonló, csak itt a gyorsulása kell

feltételezem az előző analógiájára kellene megoldani.

Az ébresztőóra 769 Hz frekvencián sípol. Egy reggel elromlik és nem lehet kikapcsolni. Ezért az órát a 4. emeleti lakásunk ablakából kidobjuk, mely a földet érésig 17 métert zuhan. (hang sebessége 343 m/s, g = 10 m/s2) Milyen gyorsan változik a hallott frekvencia (Hz/s-ben) éppen azelőtt, hogy a földön összetörik?

Előzmény: [973] Gézoo, 2012-05-18 09:06:28
[973] Gézoo2012-05-18 09:06:28

Kedves Barbi! Jó reggelt kívánok Neked is!

A végsebesség t=gyök(2s/a) ideig esik v=v0+a*t ahol most v0=0 (mert most nincs hajítás)

v=10*gyök(2*17/10)=18,44 [m/s]

Doppler f=f0*(c-v)/c ahol f0=800[Hz] és c=343 [m/s]

f=756,99 [Hz]

Előzmény: [972] szekibarbi, 2012-05-18 08:46:00
[972] szekibarbi2012-05-18 08:46:00

jó reggelt mindenkinek!

ismételten szükségem lenne néhány feladatohoz seítségre...

Az ébresztőóra 800 Hz frekvencián sípol. Egy reggel elromlik és nem lehet kikapcsolni. Ezért az órát a 4. emeleti lakásunk ablakából kidobjuk, mely a földet érésig 17 métert zuhan. (hang sebessége 343 m/s, g = 10 m/s2) Mekkora frekvencián (Hz-ben) halljuk csörögni az órát éppen azelőtt, hogy a földön összetörik?

[971] Alekszandrov2012-05-17 09:55:30

Igazad van!!!! Köszönöm szépen a zárójelet!!!

Tehát helyesen:2*m0*[1/(1-(v/c)(exp2))(exp0.5)-1] A kacsacsőrös zárójelemet lenyelte a gép...

Előzmény: [969] gorgi, 2012-05-17 09:15:22
[970] Gézoo2012-05-17 09:54:53

Nem a hiányzó zárójelen múlott az, hogy a weblap nem fogadta el az eredményt. Több módszerrel ugyanazt az eredményt kaptuk, de úgy sem volt jó.

Előzmény: [969] gorgi, 2012-05-17 09:15:22
[969] gorgi2012-05-17 09:15:22

2*m0*[1/1-(v/c)(exp2)(exp0.5)-1] nincs itt egy sajthiba? 2*m0*[1/(1-(v/c)(exp2))(exp0.5)-1]

Előzmény: [968] Alekszandrov, 2012-05-17 01:10:18
[968] Alekszandrov2012-05-17 01:10:18

Kedves Barbara!

Elképzelhető, a "hiba" abból adódik, hogy a tanárod nem használta az általam használt (Fálesz Mihály 966. hsz.-ban leírt) közelítést. Ez a közelítés mérsékelt sebességek esetén gyakorlatilag pontos. A 620 m/s jóval kisebb a fény sebességénél, ezért nyugodtan lehet a közelítést alkalmazni!

Az elméletileg is pontos megoldás: a nyugalmi tömeg változása= 2*m0*[1/1-(v/c)(exp2)(exp0.5)-1] Itt m0 a puskagolyó tömege (0.01 kg), v=620 m/s. Ha ezzel sem lesz jó, akkor már nem tudom mit lehetne még tenni az ügy érdekében!

Előzmény: [963] szekibarbi, 2012-05-16 21:35:51
[967] Alekszandrov2012-05-17 00:22:27

Én sem tudok már másra gondolni: csak ez lehet a "hiba" oka, bár szerintem v=620 m/s esetén jogosan használhatjuk az általad említett közelítést!

Előzmény: [966] Fálesz Mihály, 2012-05-16 22:24:13
[966] Fálesz Mihály2012-05-16 22:24:13

Kis x-ekre és nem túl nagy a-ra használhatod az

(1+x)a\approx1+ax

becslést. (Ezt, kicsit más formában, Geg már leírta egyszer régen.)

Kis v esetén az x=-\bigg(\frac{v}{c}\bigg)^2, a=-\frac12 választással


\frac{1}{\sqrt{1-\big(\frac{v}{c}\big)^2}} = 
\left(1-\bigg(\frac{v}{c}\bigg)^2\right)^{-\frac12} \approx 1+\frac{v^2}{2c^2}.

Előzmény: [963] szekibarbi, 2012-05-16 21:35:51
[965] Gézoo2012-05-16 21:42:52

Próbáld a ß=1/(1-(v/c)2)0,5 függvényt ugyanaz az eredmény.

Előzmény: [963] szekibarbi, 2012-05-16 21:35:51
[964] szekibarbi2012-05-16 21:38:31

majd csak pénteken, napközben is lehet? most mennem kell, mert ilyenkor a gyereknek már bőven ágyban a helye....de ahhoz kellek én is

köszönöm a sok sok segítséget

Előzmény: [963] szekibarbi, 2012-05-16 21:35:51
[963] szekibarbi2012-05-16 21:35:51

köszönöm, a gond csak annyi, hogy a számológépem amikor az arccos (1/cos)-t számolom, és utána veszem a szinuszát, egyszerűen 1-et ír ki. márpedig valami másnak kell kijönnie, mert iyen számológépet kell ahsznánlunk a vizsgán is...

Előzmény: [961] Gézoo, 2012-05-16 21:06:35
[962] Gézoo2012-05-16 21:23:08

Barbi!

Ha még a legutóbbi sem jó, akkor hívj skype-on..

vertes.geza1

Előzmény: [959] szekibarbi, 2012-05-16 20:59:10
[961] Gézoo2012-05-16 21:06:35

Hogyne!

nyugalmi tömeg m0, nyugalmi energia E=m0*c*c

v sebességnél a tömeg ß-szorosra növekszik

azaz m=m0*ß és ebben benne van a nyugalmi tömeg is.

Ezért a mozgási energiából levonjuk

igy csak m0*(ß-1) a mozgási energia tömegnövekedést okozó része v=n*c sebességre

E=m0*(sin(arccos(n))-1)*c*c

ha n=0,95 és m0=9,109e-31 valamint c*c=9e16

a mozgási energia

E=9,109e-31 * (sin(arccos(0,95))-1)*9e16

ha n=0,97

E=9,109e-31 * (sin(arccos(0,97))-1)*9e16

Természetesen ha az összes energia kell, akkor a nyugalmi is kell, olyankor nem vonjuk le

E=m0*ß*c*c

Például n=0,97 (azaz v=0,97c esetén)

E=9,109e-31 * sin(arccos(0,97))*9e16

Ha pedig a lövedékeket számoljuk a teljes energiáik:

v=620 m/s esetében a teljes energiája egy lövedéknek:

E=0,010 * sin(arccos(620/3e8))*9e16 csak a növekmény, azaz csak a mozgási energia esetén itt is levonandó a nyugalmi tömeg:

E=0,010 * (sin(arccos(620/3e8))-1)*9e16

Értelem szerűen két lövedék, kétszer ennyi energia..

Előzmény: [956] szekibarbi, 2012-05-16 20:52:26
[960] Alekszandrov2012-05-16 21:06:25

Minden tiszteletem a Tied!

Erre gondoltál? Számítsuk ki a nyugalmi energiáját (joule-ban) egy a fény sebességének 90,9-val haladó elektronnak! (elektron tömege 9,109×10-31kg, fénysebesség 3×105 km/s) A nyugalmi energia számítása: A megadott tömeget(mert az az elektron nyugalmi tömege) szorozd meg a fénysebesség négyzetével! Vigyázz, hogy azt váltsd át m/s-ba.

Előzmény: [959] szekibarbi, 2012-05-16 20:59:10
[959] szekibarbi2012-05-16 20:59:10

:)

inkább csak rettentően fáradt vagyok, mert heti 5 nap dolgozom 12 órát, gyereket nevelek, háztartást vezetek, nyelviskolába járok, készülök az államvizsgámra, meg erre a fizikára bár ezt nem nevezném készülésnek...

fáradt vagyok, és nincs energiám erre is....

Előzmény: [958] Alekszandrov, 2012-05-16 20:57:02
[958] Alekszandrov2012-05-16 20:57:02

Maximálisan egyetértek! :-)

Előzmény: [957] Gézoo, 2012-05-16 20:54:22
[957] Gézoo2012-05-16 20:54:22

Ne a fizikát utáld!

A fizika korrekt, a feladatok kiírásai - valószínűleg - szándékosan, vagy trehányságból félrevezetők. Ez nem fizika. Hanem trehányság.

Előzmény: [939] szekibarbi, 2012-05-16 20:27:59
[956] szekibarbi2012-05-16 20:52:26

nem tudom... ellenben kérhetnék levezetést arra a másik feladatra?

amikor a fénysebesség x százalékával megy az a nyomorult elektron és hány joule energia lesz?

Előzmény: [955] Alekszandrov, 2012-05-16 20:48:58
[955] Alekszandrov2012-05-16 20:48:58

Biztos, hogy a feladat kitűzője helyes eredményt adott meg?

Előzmény: [951] szekibarbi, 2012-05-16 20:43:43
[954] szekibarbi2012-05-16 20:48:57

jó, köszönöm, de nem jó a megoldás ez egy online felület, ha jó az eredmény, kapok egy zöld mosolygós fejet. eddig nem kaptam.

nem jó a megoldás.

Előzmény: [952] Alekszandrov, 2012-05-16 20:47:00
[953] Gézoo2012-05-16 20:48:05

Na ez is.. Nyugalmi energiát a nyugalmi tömegből számoljuk:

E=9,109e-31*9e16=81,981e-15=8,1981e-14 J

És nem foglalkozunk a mozgási energiával. Sem a sebességgel.

Előzmény: [949] szekibarbi, 2012-05-16 20:40:53
[952] Alekszandrov2012-05-16 20:47:00

De ugyanott vagy, azt is írhattam volna, csak részletezni akartam! Bocsánat, ha szájbarágósan írok! :-)

Előzmény: [951] szekibarbi, 2012-05-16 20:43:43
[951] szekibarbi2012-05-16 20:43:43

A FELADAT MEGOLDÁSA: HIBÁS!

mellesleg, ha valamit elosztok kettővel, és megszorzom kettővel, nem ugyanott vagyok???

Előzmény: [947] Alekszandrov, 2012-05-16 20:39:46
[950] Alekszandrov2012-05-16 20:43:05

A nyugalmi energia független attól, hogy milyen vonatkoztatási rendszert használunk!!!

Előzmény: [944] Alekszandrov, 2012-05-16 20:34:21
[949] szekibarbi2012-05-16 20:40:53

de ez másik feladat :)

ez a puskagolyós nem stimmel, meg az a másik, ahol a v=0,95

ott kijön ez az E=mc2

de a következő példa (eltérő adatokkal: Számítsuk ki a nyugalmi energiáját (joule-ban) egy a fény sebességének 90,9-val haladó elektronnak! (elektron tömege 9,109×10-31kg, fénysebesség 3×105 km/s)

itt nem jön ki...

Előzmény: [946] Gézoo, 2012-05-16 20:38:33
[948] Gézoo2012-05-16 20:40:07

Mondjuk könnyebb lenne, ha egyértelműbbek lennének a feladat kiírások :(

Előzmény: [946] Gézoo, 2012-05-16 20:38:33
[947] Alekszandrov2012-05-16 20:39:46

De én nem ezt írtam a 904.-ben! Számold ki a mozgási energiát: m0*v(exp2)/2 ezt szorozd meg kettővel, mert két elektron mozog! A kapott energiát helyettesítsd be az E=m*c(exp2) képletbe. m=E/c(exp2).

Előzmény: [943] szekibarbi, 2012-05-16 20:34:04
[946] Gézoo2012-05-16 20:38:33

A 921-ben én is ezt használtam, mert energia kellett.. abban a rendszerben ahol v=0,95c volt..

Előzmény: [943] szekibarbi, 2012-05-16 20:34:04
[945] szekibarbi2012-05-16 20:35:32

stimmelt, már beszéltem a tanárral, megoldotta, hogy normálalakba is írhassunk, mert említettem neki, hogy volt egy-két feladat, ahol a pl a 31-ik hatványon van az eredmény...

Előzmény: [942] Gézoo, 2012-05-16 20:33:08
[944] Alekszandrov2012-05-16 20:34:21

A nyugalmi energia = nyugalmi tömeg szorozva c négyzettel! Ennyi!

Előzmény: [940] szekibarbi, 2012-05-16 20:30:18
[943] szekibarbi2012-05-16 20:34:04

így

Előzmény: [941] Alekszandrov, 2012-05-16 20:31:00
[942] Gézoo2012-05-16 20:33:08

A nullák száma stimmelt?

0,000 000 000 000 024 71

Persze így írva: 0,00000000000002471

Előzmény: [937] szekibarbi, 2012-05-16 20:25:28
[941] Alekszandrov2012-05-16 20:31:00

Hogyan számoltál? Leírnád?

Előzmény: [937] szekibarbi, 2012-05-16 20:25:28
[940] szekibarbi2012-05-16 20:30:18

azt a másikat sem értem még mindig, amikor a fénysebesség x százalékával megy az a nyamvadt elektron

Előzmény: [939] szekibarbi, 2012-05-16 20:27:59
[939] szekibarbi2012-05-16 20:27:59

de, de nem jó

elegem van utálom a fizikát

Előzmény: [938] Gézoo, 2012-05-16 20:27:01
[938] Gézoo2012-05-16 20:27:01

Nocsak? Nem lett jó?

Ütközés előtt:

m=0,01*ß=0,010000000000021355555555623964519

2*m=0,020000000000042711111111247929037 kg

ütközés után állnak és ha a hőtartalmuktól eltekintünk, akkor

2*m=0,02 kg

delta m=4,271e-14 [kg]

Nem ennyit számoltunk korábban?

Előzmény: [933] szekibarbi, 2012-05-16 20:11:36
[937] szekibarbi2012-05-16 20:25:28

akkor sem jó az eredmény

Előzmény: [936] Alekszandrov, 2012-05-16 20:21:48
[936] Alekszandrov2012-05-16 20:21:48

Bocsánat 904.

Előzmény: [935] szekibarbi, 2012-05-16 20:21:16
[935] szekibarbi2012-05-16 20:21:16

de nem jó az eredmény...

Előzmény: [934] Alekszandrov, 2012-05-16 20:20:31
[934] Alekszandrov2012-05-16 20:20:31

Már a 915. hozzászólásomban leírtam!

Előzmény: [933] szekibarbi, 2012-05-16 20:11:36
[933] szekibarbi2012-05-16 20:11:36

még mindig keresem a helyes megoldást erre a feladatra

Két 620 m/s sebességgel egymással szemben haladó 10 gramm tömegű puskagolyó ütközik és összeragadnak. Mekkora lesz az ütközés utáni és előtti összes nyugalmi tömeg különbsége (kg-ban)? (c = 3×105 km/s)

[932] Gézoo2012-05-16 18:30:51

Ezt kérdeztem.

Ha a tömege v=0,95c sebességen lenne megadva akkor számítható lenne a v=0 sebességen értendő tömege.

Ezért írtam mindkét megoldást. Nézd meg figyelmesen.

Előzmény: [931] szekibarbi, 2012-05-16 18:09:31
[931] szekibarbi2012-05-16 18:09:31

igen, az annyi, de azzal nem kell foglalkozni, hogy nem fénysebességgel megy, hanem annak az x százalékával?

Előzmény: [929] Gézoo, 2012-05-16 18:01:11
[930] Gézoo2012-05-16 18:03:00

Egyébként ne aggódj! Most már nincs értelme.

Vedd lazán. Az még segíthet. Ha görcsölsz, az csak ront.

Előzmény: [927] szekibarbi, 2012-05-16 17:54:30
[929] Gézoo2012-05-16 18:01:11

(3e8)*(3e8)=9e16 vagy nem?

Előzmény: [927] szekibarbi, 2012-05-16 17:54:30
[928] Gézoo2012-05-16 18:00:23

Ha a saját tömegét emésztené fel és nem a bevezetett elektronok energiáját, akkor az égő tömege csökkenne.

A P=67 W jelölést az óránként lesugárzott 67 Wh teljesítményt szokta jelölni.

Ettől még azt is számíthatjuk, hogy az átvezetett elektronok a mozgási energiáját nézzük, és ennek a energia vesztését számítjuk tömegre számítva.

P= 67 W = 67 J/s

E=67*3*365*24*3600= 6338736000 J

m=6338736000/9e16= 0,0000000704304 [kg]

Azaz a három év alatt áthaladó összes elektron együttes tömeg vesztése 0,0000000704304 kg.

Előzmény: [926] szekibarbi, 2012-05-16 17:39:34
[927] szekibarbi2012-05-16 17:54:30

Hááát az a flikk-flakk, hogy a v=0,95c rendszerben nyugvó energiája vagy abban a rendszerben ahol v sebességgel mozog?

Az utóbbiban E=m*c*c=9,109e-31*9e16 [J]

itt a 9e16-on hogyan jött ki?

bocsánat, ha sokat foglalkoztatok mindenkit, de nem értem, és akárhogyan igyekszem, nem megy... nem is értem, miért kell ilyeneket tanulnom. el vagyok keseredve, azt érzem, hogy meg fogok bukni.

Előzmény: [921] Gézoo, 2012-05-16 15:48:29
[926] szekibarbi2012-05-16 17:39:34

Egy 67 wattos izzó 3 évnyi világítás során mennyit veszít nyugalmi tömegéből? (c = 3e5 km/s, 1 év = 365 nap)

itt gondolom e=m*c*c-vel kell számolni...

három év az 94608000s

a 67W= 67J/s.

eddig oké, és innen?

[925] szekibarbi2012-05-16 16:57:18

megvan, köszönöm.... már nem fog az agyam, államvizsgára készülök, fizika vizsgára, meg nyelvsuli...

Előzmény: [924] szekibarbi, 2012-05-16 16:56:08
[924] szekibarbi2012-05-16 16:56:08

azt csináltam, lehet elnéztem valamit, de már négyszer újraszámoltam... nekikezdek megint, hátha elkerülte valami a figyelmem

Előzmény: [923] Gézoo, 2012-05-16 16:54:18
[923] Gézoo2012-05-16 16:54:18

Kedves Barbi!

a 910-ben leírt sorokba helyettesítsd be az új adatokat.

Gyakorlásnak sem rossz.

(És hagy ne kelljen nekem behelyettesítenem.)

Előzmény: [922] szekibarbi, 2012-05-16 16:50:17
[922] szekibarbi2012-05-16 16:50:17

tudom, hogy ezt az előbb vezettétek le, de rossz a megoldásom??

Két egyforma, gömb függ egy-egy azonos pontba felfüggesztett 17-17 cm hosszú fonálon. A gömbök tömege 40 gramm, töltésük azonos nagyságú és előjelű. Egyensúlyi helyzetben a fonalak 7°-os szöget zárnak be a függőlegessel. Mekkora a gömbök töltése (nC-ban)? (k = 9×109 Nm2/C; g = 10 m/s2)

ez nem 136,8nC???

A=0,0207m B=0,1687m

2F=0,09816N Q=1,3684*10-7C-> 136,84nC

egyszerűen nem fogadja el

[921] Gézoo2012-05-16 15:48:29

Hááát az a flikk-flakk, hogy a v=0,95c rendszerben nyugvó energiája vagy abban a rendszerben ahol v sebességgel mozog?

Az utóbbiban E=m*c*c=9,109e-31*9e16 [J]

Az előbbiben E=m*c*c*(ß-1)

E=m*c*c*2,2026

(ß=1/sin(arccos(v/c))=1/sin(arccos(0,95))=3,203 )

Előzmény: [918] szekibarbi, 2012-05-16 15:36:45
[920] szekibarbi2012-05-16 15:42:58

semmi gond, így is többet értettem meg itt, mint az órán

Előzmény: [919] Gézoo, 2012-05-16 15:41:13
[919] Gézoo2012-05-16 15:41:13

Látod én is tévedhetek! Elcseréltem a töltéseket.. Bocs!

q1=6e-6

q2=13e-6

q3=-3e-6

R=2,3-r

k*q1*q3/r2=k*q2*q3/R2

q1/r2=q2/R2

R2=(2,3-r)*(2,3-r)=2,32-2*2,3*r+r2

R2/r2=q2/q1=

R2/r2=13e-6/6e-6=

R2/r2=13/6=

(5,29-4,6*r+r2)/r2=13/6

Előzmény: [917] szekibarbi, 2012-05-16 15:19:42
[918] szekibarbi2012-05-16 15:36:45

Számítsuk ki a nyugalmi energiáját (joule-ban) egy a fény sebességének 95százalékával haladó elektronnak! (elektron tömege 9,109×10-31kg, fénysebesség 3×105 km/s)

ezt a feladatot az e=mc2-el kell számolni, vagy van valami flikk-flakk benne?

[917] szekibarbi2012-05-16 15:19:42

tudni tudom, csak nem jó az eredmény, ami kijött...

Előzmény: [916] Gézoo, 2012-05-16 14:48:21
[916] Gézoo2012-05-16 14:48:21

Fogalmam sincs..

Azzal is tanulhatsz, ha azt részletezed, hogy mit számoltál!

Én, úgy gondolom, hogy r=x6

R=2,3-r

Fr=k*3*6*10-12/r2 = FR=k*13*6*10-12/(2,3-r)2

azaz:

3/r2=13/(2,3-r)2

Tudod folytatni az "r"-re rendezést?

Előzmény: [914] szekibarbi, 2012-05-16 14:23:16
[915] Alekszandrov2012-05-16 14:48:19

A Q1 töltés által a Q3-ra kifejtett erő egyenlő a Q2 által a Q3-ra ható erővel!

Tehát Q1*Q3/(2,3-x)exp2=Q2*Q3/xexp2 Ez x-re nézve másodfokú egyenlet, ezt kell megoldanod!

Előzmény: [914] szekibarbi, 2012-05-16 14:23:16
[914] szekibarbi2012-05-16 14:23:16

Egy egymástól 2.3 méter távolságra fekvő Q1 = 13 mikrocoulomb és Q2 = 6 mikrocoulomb töltésű pozitív töltések közé behelyezünk egy Q3 = -3.0 mikrocoulomb negatív töltést. Milyen távol (m-ben) van az egyensúlyi helyzete a Q2 töltéstől?

jó a levezetés?

f1->2= k[(q1*q2)/x2] f2->3= k[(q2*q3)/y2]

egyszerűsítés után

q2/x2 = q3/y2

y=67,9x

[913] Alekszandrov2012-05-16 13:34:29

Kedves Barbara!

1. Figyelembe véve az elektron tömegét és töltését, a gravitációs tér hatását nyugodtan elhanyagolhatjuk! 2. A relativisztikus hatásokat is elhanyagolhatjuk, hiszen az elektron sebessége századrésze a fény sebességének! 3. Az elektron vízszintes irányban egyenes vonalú egyenletes mozgást végez, míg függőlegesen lefelé (azért lefelé, mert a térerősség vektor iránya definíció szerint a pozitív töltésre ható erő irányával egyezik meg) egyenes vonalú egyenletesen gyorsuló mozgást, melynél a függőleges irányú kezdősebesség nulla!

Jelölések: E:= elektromos térerősség (222 N/C) v:= az elektron sebessége a belépés előtt (3,5*10(exp6)) x:= az elektron vízszintes elmozdulása az elektromos térben (12 cm= 0,12 m) m:= az elektron tömege (adott) e:= az elektron töltése (adott) a:= az elektron függőleges irányú gyorsulása (közbenső adat) t:= az elektromos térben töltött idő (közbenső adat) y:= az elektron függőleges elmozdulása (kérdezik)

Megoldás: A kétféle mozgást egymástól függetlenül vizsgálhatjuk! y= (a/2)*t(exp2) hiszen álló helyzetből t idő alatt ekkora elmozdulást végez egyenes vonalú egyenletesen gyorsuló mozgással az anyagi pont. Amint láthatod a kérdés megválaszolásához a-ra és t-re van szükséged! A t időt kiszámíthatod a vízszintes irányú egyenes vonalú egyenletes mozgásból: t=x/v

Az E térerősségű homogén elektromos tér az e töltésű elektronra F= E*e nagyságú erővel hat. Newton törvénye: F=m*a Ebből: a= F/m= E*e/m. Már kész is vagyunk! :-) (cm-be még át kell váltani!!!)

Előzmény: [909] szekibarbi, 2012-05-16 12:11:01
[912] Gézoo2012-05-16 13:07:35

Oké.. de azért mindig gondold át, mert én is tévedhetek!

Szóval E=F/q = 222 [N/C]

F=E*q=222*1,602*10-19=0,0000000000000000355644 [N]

ekkora erő gyorsítja az

m=9,11*10-31 [kg] tömeget

s=0,12 [m] úton, miközben az

v=3,5*106 [m/s] sebességgel halad

t=s/v=0,12/3,5*106 [s] ideig:

t= 3,42857E-08 [s]

azaz a gyorsulást okozó F erő is ennyi ideig hat

a=F/m=222*1,602*10-19/9,11*10-31=3,904*1013m/s2

h=(1/2)a*t2=0,022945288 [m]

hcm=2,2945288 [cm]

Az igazság szerint a v=3,5e6 m/s-mellett figyelembe kellene venni a 0,12 m rövidülését az elektron rendszerében. De gondolom, hogy ebben a feladatban erről nincs szó.

Előzmény: [909] szekibarbi, 2012-05-16 12:11:01
[911] szekibarbi2012-05-16 12:29:43

köszönöm, így már érthető... a gond, hogy az órán egy számolást sem vezettünk le, én az öreg fejemmel nem értek a fizikához, soha nem is értettem... viszont a vizsgán csak számolás lesz. és nem tudom őket megoldani, ellenben így, hogy le van vezetve kezdem érteni.

Előzmény: [910] Gézoo, 2012-05-16 12:17:42
[910] Gézoo2012-05-16 12:17:42

G=0,037*10=0,37 [N]

A/0,15=sin(3°)

B/0,15=cos(3°)

A=sin(3°)*0,15

A=0,00785 [m]

B=cos(3°)*0,15

A/B=F/G --> F=G*A/B

F=G*sin(3°)*0,15/(cos(3°)*0,15)

F=G*sin(3°)/cos(3°)

F=0,0194 [N]

2*F=0,0388 [N]

R=2*A=0,0157 [m]

R2=0,00024649 [m]

Q=(F*R2/k)0,5

Q=2,305*10-8 [C]

Nekem ez jött ki. Gondold át és ellenőrizd, mielőtt begépeled. (0,000 000 023 05 )

Előzmény: [907] szekibarbi, 2012-05-16 11:03:24
[909] szekibarbi2012-05-16 12:11:01

Egy elektron függőlegesen felfelé mutató homogén, 222 N/C nagyságú elektromos térbe lép 3.5×106 m/s nagyságú sebességgel vízszintes irányban. Az elektromos térből való kilépés előtt 12 cm-t repül vízszintes irányban. Mekkora a belépési és a kilépési pontok közötti távolság (cm-ben) függőleges irányban? me = 9,11×10-31kg; e = 1,602×10-19C; k = 9×109Nm2/C

[908] szekibarbi2012-05-16 11:15:19
Előzmény: [907] szekibarbi, 2012-05-16 11:03:24
[907] szekibarbi2012-05-16 11:03:24

lehet hülyének néznek, de nem megy... feltételezem itt a szinusztételt kell alkalmazni. jól gondolom? így a/sinalfa=b/sinß=c/singamma

ebből nekem az lett, hogy

0,15/sin90=b/sin15 = 0,0388 mivel ez csak a "fele" így a vízszintes erő: 0,07765N

jól gondolom?

Előzmény: [906] Gézoo, 2012-05-16 10:38:29
[906] Gézoo2012-05-16 10:38:29

Kedves Barbi!

Van két súlyerőd, egyenként F=m*g= ? A súlyerők függőlegesek, ezért a 3°-os, 0,15 m hosszú "átfogó"-ból kiszámíthatod (az erők arányaival egyező,) a függesztési háromszög oldalainak arányát. Ebből pedig a a vízszintes erők nagyságát.

A vízszintes erők ha rugó lenne a gömbök között a rugót összenyomnák. Azaz a rugóerővel azonos nagyságú F=k*Q2/R2 erő hat a gömbök között.

Ebből pedig Q=(F*R2/k)0,5 függvénnyel megkapod a Q töltés nagyságát.

Előzmény: [905] szekibarbi, 2012-05-16 10:24:51
[905] szekibarbi2012-05-16 10:24:51

Két egyforma, gömb függ egy-egy azonos pontba felfüggesztett 15-15 cm hosszú fonálon. A gömbök tömege 37 gramm, töltésük azonos nagyságú és előjelű. Egyensúlyi helyzetben a fonalak 3°-os szöget zárnak be a függőlegessel. Mekkora a gömbök töltése (nC-ban)? (k = 9×109 Nm2/C; g = 10 m/s2) ???

[904] Alekszandrov2012-05-15 20:29:54

Kedves Barbara!

Szerintem számold ki az ütközés előtti összes mozgási energiát (mindkét lövedék sebessége 620 m/s a Földhöz képest)! Az ütközés során ezt az energiát nyeli el a két test közelítőleg, tehát az összes mozgási energiát helyettesítsd be az E=m*c(exp2) képletbe és máris megkapod a nyugalmi tömegek változását a folyamat során.

Előzmény: [891] szekibarbi, 2012-05-14 20:20:07
[903] Gézoo2012-05-15 10:08:45

Kedves Lajos bácsi! Egyetértek veled és a cikk írójával is. Sőt! Einstein áltrelje is erről szól. Az elgörbülő "végű" lézer szike esetében is valószínűleg valami ilyesmiről lehet szó, ahol talán az elhajlást a vágás környezetében lévő sejtek anyaga okozhatja. Ezzel együtt is jó lenne többet tudni a módszerről.

Előzmény: [902] Lajos bácsi, 2012-05-15 09:21:08
[902] Lajos bácsi2012-05-15 09:21:08

A fény görbítésének egyik lehetőse a fény terjedési sebességének megváltoztatásából adódhat. Erről szól egy okfejtés Tassi Tamás írásában: itt.

Idézet a cikkből: "Ezáltal a hullámnak a felső része mindig gyorsabban halad ezért aztán folytonosan kanyarodik."

A görbítéshez tehát eltérő anyag vagy valamilyen erőtér (gravitáció) szükséges, de hogy hogyan, a kérdés még nyitott.

Előzmény: [901] Gézoo, 2012-05-15 08:12:41
[901] Gézoo2012-05-15 08:12:41

Kedves Alma!

Ha nem ismerted ezt a levezetést, akkor milyen levezetésről írtad azt, hogy "Le tudom vezetni."

Előzmény: [887] Alma, 2012-05-14 19:54:11
[900] Gézoo2012-05-15 08:11:27

Igazából ez a lövedékes feladat messzebbre mutató ismereteket igényelne. Például függvény szerint adjuk össze a két 620 m/s sebességet abban a rendszerben ahol kilőtték a lövedékeket. A lövedékek összes energiái pedig: függvény szerint képezendők és ennek a kétszerese osztandó a fénysebesség négyzetével a mozgó tömeg nagyságának meghatározásakor. Azaz bármelyik megoldással próbálkozunk az valamilyen szempontból hibás lesz, ha a tanárod nem ugyanazt a megoldási utat választotta.

Előzmény: [897] szekibarbi, 2012-05-14 21:01:24
[899] Gézoo2012-05-15 08:06:21

Sajnos keveset írnak róla. Amit írtak abból pedig úgy tűnik, hogy készülnek a Jedi kardok. :-)

Előzmény: [898] Zilberbach, 2012-05-14 21:04:11
[898] Zilberbach2012-05-14 21:04:11

http://www.hir24.hu/tudomany/2012/05/14/fenycsavarassal-mutenenek/

Tudja valaki, hogyan működik ez?

[897] szekibarbi2012-05-14 21:01:24

hát ez nem a legjobb eredmény :) lehet megint el van írva a feladat, ma csak 5 feladatot javítattam ki a tanárral...

Előzmény: [896] Gézoo, 2012-05-14 20:56:50
[896] Gézoo2012-05-14 20:56:50

Az ütközés előtt:

m=2*m0/(1-(620/3e8)2)0,5

m=0,02000000000004271 [kg]

Ütközés után állnak, így m=2*0,01 [kg]

különbözet=4,271111111124793e-14 kg

(0,000 000 000 000 042 711111111247929 [kg] )

Előzmény: [895] szekibarbi, 2012-05-14 20:31:34
[895] szekibarbi2012-05-14 20:31:34

ez a szöveg ctrl-c ctrl-v vel van...

Előzmény: [894] Gézoo, 2012-05-14 20:30:01
[894] Gézoo2012-05-14 20:30:01

Esetleg ha pontosan idéznéd a kiírt szöveget, az sokat segíthetne. (Bár igaz, a múltkor is félreérthető szöveg volt a GPS órájával.)

Előzmény: [893] szekibarbi, 2012-05-14 20:28:21
[893] szekibarbi2012-05-14 20:28:21

hát ez az bár szerintem inkább az utolsó

Előzmény: [892] Gézoo, 2012-05-14 20:25:38
[892] Gézoo2012-05-14 20:25:38

Melyik rendszerben nézzük? Ott ahol az egyik áll és a másik 1240 m/s sebességű, ott ahol mindkettő 620 m/s sebességű?

Vagy úgy értsem, hogy a golyók közötti relatív sebesség összesen 620 m/s ?

Előzmény: [891] szekibarbi, 2012-05-14 20:20:07
[891] szekibarbi2012-05-14 20:20:07

kérhetnék ehhez is levezetést?

Két 620 m/s sebességgel egymással szemben haladó 10 gramm tömegű puskagolyó ütközik és összeragadnak. Mekkora lesz az ütközés utáni és előtti összes nyugalmi tömeg különbsége (kg-ban)? (c = 3×105 km/s)

Előzmény: [886] Gézoo, 2012-05-14 19:44:58
[890] Gézoo2012-05-14 20:17:44

C' út, azaz a tükrök távolsága ezen az ábrán "r" betűvel van jelölve, a C út "Z" betűvel, a V út pedig "k"-val: Felül a kék tükrök síkja, IRa és IRb a két tükrös rendszer mozgása, amelyből célszerűen csak az egyik oldal K-Z-R háromszögét vegyük figyelembe. (Választhatsz, hogy melyiket :)

( A narancs és a piros -- a jobb oldalon, -- valamint a kék és zöld -- a bal oldalon, -- vektorok végei a haladó fotonokra mutatnak rá.)

Előzmény: [887] Alma, 2012-05-14 19:54:11
[889] szekibarbi2012-05-14 20:16:06

de mégis :) bocsi csak elírtam egyel a tizedesjegyet :)

Előzmény: [888] szekibarbi, 2012-05-14 20:15:01
[888] szekibarbi2012-05-14 20:15:01

grammban, de nem jó... :(

Előzmény: [886] Gézoo, 2012-05-14 19:44:58
[887] Alma2012-05-14 19:54:11

Egy ábrát kérhetek?

Előzmény: [885] Gézoo, 2012-05-14 19:37:40
[886] Gézoo2012-05-14 19:44:58

"A trinitro-toluol (TNT „tonnáját” (általában kilotonna, megatonna, gigatonna nagyságrendben) az energia mértékegységeként is szokás használni, különösen nukleáris fegyverek pusztító erejének kifejezésére.

A TNT fűtőértéke 4,184 MJ/kg (vagy 1 kalória per milligramm). Így, 1 kilotonna TNT = 4,184 TJ, 1 Mt TNT = 4,184 PJ."

1 Mt= 1 000 000 t = 1 000 000 000 kg

5 Mt = 5 000 000 000 kg

E= 5 000 000 000 * 4,184 [MJ]

E= 5 000 000 000 * 4 184 000 [J]

E= 20 920 000 000 000 000 [J]

m=E/c2

m=20 920 000 000 000 000 / 9e16 [kg]

m= 0,2324 [kg]

Ezek után azt is nézd meg, hogy a választ milyen mértékegységben kell begépelned.

Előzmény: [880] szekibarbi, 2012-05-14 19:04:21
[885] Gézoo2012-05-14 19:37:40

Kedves Alma!

Vezessük le a fényórában lévő fényutak hosszából.

Legyen a tükrök közötti fényút hossza C' az átlón haladó fény úthossza C és a tükör V úton haladjon v sebességgel.

Azaz C2=C'2+V2 miután a fényutak aránya egyezik a sebességek arányáival:

c2=c'2+v2 és a derékszögű háromszöget alkotnak, amelyre Püthagorasz-tétele érvényes:

c'2=c2-v2

c'=(c2-v2)0,5

a tükrök között haladó fény sebessége. A valós c fénysebesség és a c' fénysebesség aránya:

\gamma=c/c'=c/(c2-v2)0,5

1/\gamma=c'/c=(c2-v2)0,5/c

1/\gamma=c'/c=((c2-v2)/c2)0,5

1/\gamma=c'/c=((c2/c2-v2/c2))0,5

c2/c2=1 azonossággal:

1/\gamma=c'/c=((1-v2/c2))0,5

valamint a megszokott alakra rendezéssel:

\gamma=c/c'=1/((1-(v/c)2))0,5

Tehát mi ez a c' sebesség és milyen arány a c/c' aránya ?

Előzmény: [882] Alma, 2012-05-14 19:20:41
[884] Alma2012-05-14 19:36:48

Szerintem \Deltam=m-m0 a kérdés.

Előzmény: [880] szekibarbi, 2012-05-14 19:04:21
[883] Gézoo2012-05-14 19:23:30

m=9,109*10-31/(1-0,9092)0,5 = 9,109*10-31/0,416796 = 2,185481*10-30 [kg]

(=0,000 000 000 000 000 000 000 000 000 002 18 kg)

Előzmény: [877] szekibarbi, 2012-05-14 18:20:36
[882] Alma2012-05-14 19:20:41

Kedves Gézoo!

Le tudom vezetni. Nem tudom milyen sebességek arányítására gondolsz.

Előzmény: [879] Gézoo, 2012-05-14 19:00:55
[881] szekibarbi2012-05-14 19:12:11

mert számomra ez lenne az eredmény:

5 Mt = 5e9 g nos akkor ez 2,0935e10 J

és mivel

e=mc2

így számomra m= 0,232611111

Előzmény: [880] szekibarbi, 2012-05-14 19:04:21
[880] szekibarbi2012-05-14 19:04:21

okés, akkor megint rossz a kiírás... :)

és ez? :)

Egy 5.0 Mt-ás bomba felrobbantása során mennyi az átalakult tömeg (g-ban)? (1 t TNT – 109 cal, 1 cal = 4,187 J)

Előzmény: [878] Gézoo, 2012-05-14 18:56:54
[879] Gézoo2012-05-14 19:00:55

Kedves Alma!

Rövidítsünk tovább, adva van a Lorentz transzformáció gammája: \gamma=1/(1-(v/c)2)0,5 alakban. Le tudod vezetni? Tudod milyen sebességek arányítását végezzük vele?

Előzmény: [875] Alma, 2012-05-14 13:56:18
[878] Gézoo2012-05-14 18:56:54

Kedves Barbi!

Jó a függvény, helyettesíts be és megkapod az eredményt.

Előzmény: [876] szekibarbi, 2012-05-14 17:43:50
[877] szekibarbi2012-05-14 18:20:36

pontosítok, lemaradt a kérdés fele...

Számítsuk ki a tömegnövekedését (kg-ban) egy a fény sebességének 90.9 százalékával haladó elektronnak! (elektron tömege 9,109×10-31kg, fénysebesség 3×105 km/s)

[876] szekibarbi2012-05-14 17:43:50

Lenne megint néhány kérdésem... segítenétek?

Számítsuk ki a tömegnövekedését (kg-ban) egy a fény sebességének 91.0

ha jó gondolom, ennek a megoldása: m= mo/(gyök1-(v/c)négyzet)

[875] Alma2012-05-14 13:56:18

Kedves Gézoo!

Egy B test relatív sebessége egy A testhez viszonyítva definíció szerint B sebessége abban a koordináta-rendszerben, melyben A áll. Ez invariáns, nem függ a szemlélő koordinátarendszerétől az értéke (legalábbis a nagysága). Jól látod, valóban nem invariáns s'/t', sem pedig s'*t'.

Amit eddig beláttunk (miután szépen korrektül definiáltad nekem s, t, s' és t' mennyiségeket), hogy

s'/t'=s/t.

Még mindig nem írtad le, miért lesz

s'*t'=s*t,

illetve nem vezetted le tisztán, hol az ellentmondás a fénysebesség állandóságával. Kíváncsian várom okfejtéseid.

Előzmény: [837] Gézoo, 2012-05-10 05:45:06
[874] Gézoo2012-05-14 12:58:53

A lapos Föld teóriáját éppen úgy, mint a Heliocentrikus világképet vagy a mindenséget kitöltő egyetlen galaxis létét szintén híres emberek hirdették.

Felejtsük el, hogy ki mondta és csak arra figyeljünk ami mérésekkel igazolt tény:

rel Doppleres vagy nem rel. Doppleres az érzékelt frekvencia amit mindkét fél a másiktól jövő villanás sorozatként tapasztal?

Mert ha tudsz olyan kísérletről amelyben a rel. Doppler nem érvényesül, akkor szólj! Engemet is érdekelne, hogy miért nem érvényesült ott!

Ha pedig mindig érvényesül, akkor más hatás nem jelent meg a rel. Doppleren kívül.

Mert ha megjelent volna, akkor nem a rel.Dopplert mértük volna, hanem a két hatás eredőjét.

Előzmény: [873] Zilberbach, 2012-05-14 11:54:21
[873] Zilberbach2012-05-14 11:54:21

Jól érzed, tényleg nem vagyok meggyőzve. Természetesen egyetlen, a fizikához valamennyire is értő ember sem tagadja a Doppler-hatás jelenségét, az úgy működik ahogy Te leírtad. Azonban sokan állítják, hogy ezen fölül még létezik a sebesség okozta időlassulás is, amin az ikerparadoxon alapul. Ez valóban nehezen fölfogható és kezelhető jelenség, de vannak erre utaló jelek.

Előzmény: [872] Gézoo, 2012-05-14 09:23:39
[872] Gézoo2012-05-14 09:23:39

Kedves Zilberbach!

Úgy érzem, hogy nem volt eléggé meggyőzőek az iker-paradoxonnal kapcsolatos eddigi magyarázataim.

Azt írtad, hogy:"Próbáld meg elképzelni: a gyorsan utazó testvérnél sokkal kevesebb villanás tud megtörténni, mint az itt maradt testvérnél. Ez az iker-paradoxon lényege: lelassul nála az idő."

Nos, nem csak elképzeljük, de fizikailag mérjük is a kevesebb villanást. Nyilván tudod, hogy a megszámlált villanásoknál kevesebb villanást egyik iker sem küldhet a másik felé. A megszámláltak pedig a relativisztikus Doppler függvénye szerinti sűrűséget és számot mutatják. Nem többet és nem kevesebbet.

Bizonyára azt is tudod, hogy a fény elöl az oda és vissza út során nem tud "megszökni" egyik iker sem, mert ha még lehagyhatná is a másik iker felől érkező fényt, akkor is találkozna vele a visszaút során.

Tehát a megszámlált villanások garantálják a mérés helyességét.

A mérési tapasztalatból pedig azt is tudjuk, hogy ha két villogó megfigyeli a másikat, akkor a villanások száma egyezik mind addig amíg egyenletes sebesség mérhető köztük és egyenlő gravitációs térerősségben vannak.

Azaz bármilyen más eredményt adó gondolat kísérlet, csak a fizikai tényekkel ellentétes eredményt adhat.

És miután akár egyetlen mérési módszer is elegendő egy gondolat kísérlet helyességének megcáfolásához, a rel.Doppleres mérés ezt a cáfolást elvégezte és elvégzi minden nap millió számra.

Azok a neves tudósok pedig, akik ennek a kísérleti bizonyíték sornak ellenére állítják, hogy az iker-paradoxon létező jelenség, csak azt bizonyítják az állításukkal, hogy nem rendelkeznek a szükséges és elégséges minimális ismerettel az ikerparadoxon tekintetében. Ez pedig felveti azt a gyanút, hogy a relativitás többi részét is az iker-paradoxonhoz hasonló tájékozatlansággal véleményezték.

Előzmény: [868] Zilberbach, 2012-05-13 20:53:53
[871] Gézoo2012-05-13 22:44:38

Sajnos, az erőméréssel még csak a 10-14 [N] nagyságrendnél tartunk. Idő méréssel pedig a hordozható "atomórákkal" a femto és atto másodperceknél. Azaz elegendően hosszú időszakasz alatt a különbség felhalmozódása további nagyságrendekkel javítja a különbség mérhetőségét. Egy nap például 24*3600=86 400 másodpercével mindjárt négy nagyságrenddel pontosabb különbség mérést tesz lehetővé. Ha pedig a gamma rezonátorokat (lézereket) is sikerül stabilizálni, akkor pedig 10-21 és 10-24 [sec] elérésével újabb nagyságrendekkel pontosabb atomórákat fognak készíteni. Ezekkel pedig már simán elérhető a 10-27 [sec] felbontás, 10-30 [sec] felbontással pedig mm-es magasságkülönbség okozta gravitációs tárerősség különbséget is ki tudunk majd mérni.

Előzmény: [867] Zilberbach, 2012-05-13 19:32:01
[870] Gézoo2012-05-13 22:30:51

A hullám "esése" felvetésed is nagyon érdekes.

Einstein korában a kronométerek heti pontossága elérte az egy másodpercet. Nyilván abban az időben a számítások pontossága sem sokkal haladta meg a pár tizedes jegynyit. Ezért igazából nem fair a mai több száz jegy pontosságú számoló eszközökkel ismert eredményeket összevetni azzal, amit Einstein ismerhetett.

Az ikrek kérdésében is jó a felvetésed! Valóban ha lassabb lenne valamelyikük a másiknál akkor kevesebbet sugározhatna..

Na de a rel.Doppler adta frekvencia mérést széles körben használjuk, az autókban éppen úgy mint a pici és nagy repülőkben, a rendőrségi sebességmérőkben, vagy akár a meteorológiai méréseknél. Sok millió alkalmazás ellenőrzött eredményéből tudjuk, hogy a távolodó lassult frekvenciája éppen úgy lassult mint a távolodó ikertesó órájának lassultságából adódó frekvencia.

A közeledőnél úgyszintén szimmetrikus a gyorsult frekvencia. Ezért biztonsággal kijelenthető az, hogy egyik iker sem tud úgy lelassulni, ahogy azt mi már mérések milliárdjaiból ne ismernénk.

Azaz a lentebbi "paradoxon cáfolat" fizikai tényen alapul.

A gyorsulás hatásának kitett test akár szabadon esik, vagy áll a gravitációban, valamint erőhatás alatt áll a gyorsuló rakétában egyaránt ugyanazon időlassulást tapasztalja. De ez teljesen más okból történik mint az egyenletes (állandó) sebesség okozta varphi=arccos(v/c) ami tetejében teljességgel szimmetrikus jelenség, a gyorsulásos időlassulás teljességgel aszimmetriájával szemben.

Abban igazad van, hogy neves emberek hirdetik az iker paradoxon létét, mint ahogyan nagyon neves emberek hirdették eddig már sok minden létét. A fizikai bizonyíték ellent mond a tekintélyükkel alátámasztott kijelentéseiknek.

Előzmény: [868] Zilberbach, 2012-05-13 20:53:53
[869] Gézoo2012-05-13 22:16:10

Kedves Zilberbach!

Nos, ha két óra egymás felett 2 [m] magasság különbséggel 10-27 nagyságrendű különbséggel méri az időt másodpercenként,

g=10m/s2 gravitációban akár áll, akár szabadon esik, akkor jól mérhető a gravitációs időlassulás hatása.

Ha a h=2 m magasságot t=2/3e8 sec alatt teszi meg a fény, akkor a szabadon eső szoba, sebesség változása

\Deltav=a*t Ezen sebesség különbségen a fentről lefelé irányban a rel.Doppler

\Deltaf=((c-v)/(c+v))0,5 frekvencia változása \Deltaf=2,2222222222222225308641974629423*10-16 [Hz]

a lenti és fenti órajel frekvencia közötti különbség pedig

\Deltaf=g*h/c2=2,2222222222222222222222222222222*10-16 [Hz]

Tehát valóban nagyon kicsiny a két frekvencia különbsége, mindkét mérési módszerrel, de létező.

Főleg ha azt is tudjuk, hogy az alsó óra a lassabb ennyivel, és fentről ehhez az órajelhez a számított frekvencia változással pedig növelt frekvenciájú jel érkezik. Vagyis a változás duplája a különbözet.

Előzmény: [866] Zilberbach, 2012-05-13 19:06:00
[868] Zilberbach2012-05-13 20:53:53

Kedves Gézoo!

Próbáld meg elképzelni: a gyorsan utazó testvérnél sokkal kevesebb villanás tud megtörténni, mint az itt maradt testvérnél. Ez az iker-paradoxon lényege: lelassul nála az idő. Ez első hallásra ellenkezik a józan ésszel (sőt másodikra is) - de ha tényleg létezik az iker-paradoxon akkor mégis ez az igazság. Ha nem létezik az iker-paradoxon, akkor igazad van. Sok neves fizikus állítja, hogy létezik az ikerparadoxon. Azt azonban nyugodtan állíthatjuk, hogy ha létezik, akkor viszont egyáltalán nem szimmetrikus a viszony a fölgyorsított és az állva maradt rendszerek között, legalábbis az idő tekintetében nem az.

Előzmény: [865] Gézoo, 2012-05-13 14:34:17
[867] Zilberbach2012-05-13 19:32:01

Én még a legreálisabban kivitelezhető módszernek azt látnám, ha sikerülne mérni, hogy a mennyezetnél valamivel gyengébb a gravitációs tér, mint a padlónál.

Előzmény: [866] Zilberbach, 2012-05-13 19:06:00
[866] Zilberbach2012-05-13 19:06:00

Úgy gondolom, hogy a fény is szabadesést végez gravitációs térben mint minden más. (Gondolom ez nem nagyon lep meg, hiszen mindig is amellett érveltél, hogy a fény az foton-részecske.) Természetesen emellett még fénysebességgel halad is pályáján. A mi világunkban ez lényegesen nagyobb sebesség mint a szabadesésének a sebessége, ezért ez a hétköznapi életben nem tűnik föl. Én úgy gondolom hogy egy átlagos méretű liftben a jelenlegi technikai lehetőségekkel nem lehet megállapítani hagy szabadon esik, az űrben lebeg, vagy tehetetlenségi pályán halad a lift. Persze ez egy gondolat-kísérlet, ezért kicsit furcsán hat a mai technikai lehetőségeket emlegetni, de vegyük úgy, hogy ez a realitás allegóriája. (Természetesen emellett az a véleményem, hogy ugyanúgy jogos a fényt elektromágneses hullámnak tekinteni, és akkor jogos a kérdés: hogy létezik hogy "lefelé esik a hullám"?) Úgy tudom a mai technikai lehetőségekkel is sikerült kimérni: a föntről lefelé irányított fénysugár gyorsabban teszi meg ugyanazt az utat, mint a lentről fölfelé irányított.

Előzmény: [864] Gézoo, 2012-05-13 14:31:44
[865] Gézoo2012-05-13 14:34:17

Nos, amikor az ikrek mindketten megszámlálják a testvérüktől érkező villanások számát, akkor egymás mellé visszaérve nem lehet különbség a megszámlált és az óráik által mutatott számlálóállások között. Vagy másként gondolod?

Előzmény: [863] SmallPotato, 2012-05-12 12:21:52
[864] Gézoo2012-05-13 14:31:44

Hú, igazad van! Nagyon jól látod!

"Esik lefelé" a fény is.

Szóval úgy gondolod, hogy a szabadon eső és a tehetetlenségi pályán haladó szoba belsejében nem lehetne megkülönböztetni egymástól a két rendszert?

Előzmény: [862] Zilberbach, 2012-05-12 12:20:04
[863] SmallPotato2012-05-12 12:21:52

"Fel sem merülhetett benne [Einsteinben], hogy valaha majd például én a számlálós módszerrel bemutatom az ikerparadoxon lehetetlenségét."

Azért - mindent összevéve - én úgy gondolom, hogy ez szerencse.

Előzmény: [857] Gézoo, 2012-05-12 10:51:41
[862] Zilberbach2012-05-12 12:20:04

Én úgy látom hogy, a példádban nem kell, és nem is tudunk áttérni egy másik vonatkoztatási rendszerre: mindig a lift marad a vonatkoztatási rendszer, és az együtt esik a fénnyel, vagyis nem lesz érzékelhető frekvencia-változás a mennyezeti lámpa fényében. A fény ugyanúgy gyorsul a padló irányába mint a lift többi része, tehát ez belülről nem lesz érzékelhető. (Az egyszerűség kedvéjért tételezzük föl a "legkonvencionálisabb" esetet: vagyis a lift és vele minden, a padló felé esik).

Előzmény: [861] Gézoo, 2012-05-12 11:57:25
[861] Gézoo2012-05-12 11:57:25

Igazából sok módszerrel megkülönböztethető a szabadesés és a tehetetlenségi pályán haladás.

Egy példa: villant a liftben a fenti lámpa. Ha tehetetlenségi pályán halad a lift akkor nem végez sebesség változást, azaz amíg a lift aljára érkezik a fény továbbra is inerciális pályán haladva a frekvenciája nem változik.

Ha pedig gyorsul, akkor a fény utazási ideje alatt átlép egy másik IR-be, miután a gyorsulás az a sebesség megváltozását jelenti vagyis a fény színe megváltozik. Ha a lámpa felőli oldala felé gyorsul a lift akkor kékül, ha a padló felé akkor vörösödik a lámpa fénye a padlót elérve.

Így egy egyszerű frekvencia méréssel vagy színméréssel megállapítható a gyorsulás léte.

És a te ölteled is jó.. Valóban, a részecskék beérkezéséből (ezzel az élettartamából) is megállapítható a gyorsulás forrása.

Előzmény: [860] Zilberbach, 2012-05-12 11:46:08
[860] Zilberbach2012-05-12 11:46:08

Egy kis kétely azért mocorog még bennem. Ugyanis 0,9999 C-vel a föld felé "hasító" mezon, mellesleg még szabadon-esik is. A relativitás-elméletben olyanokat is lehet olvasni, hogy egy szabadon eső liftben nem lehet kimutathatni a gravitációs tér hatásait, tehát nem tudjuk megállapítani, hogy szabadon esünk egy gravitációs térben, vagy lebegünk mindentől távol, az űrben. Ha az igaz amit írtál, akkor meglenne a módszer hogy eldöntsük szabadon esünk-e egy gravitációs térben, vagy kint lebegünk az űrben: keltsünk a liftben különböző irányokban gyorsan mozgó, de nagyon rövid életű részecskéket, és ha ezek élettartam változásait elemezzük, akkor még a gravitációs erő irányát is megállapíthatjuk.

Előzmény: [857] Gézoo, 2012-05-12 10:51:41
[859] Gézoo2012-05-12 11:44:22

Szívesen!

Eléggé érthetően írtam? Vagy esetleg maradt benned bizonytalanság?

Előzmény: [858] Zilberbach, 2012-05-12 11:23:56
[858] Zilberbach2012-05-12 11:23:56

Köszönöm a választ!

Előzmény: [857] Gézoo, 2012-05-12 10:51:41
[857] Gézoo2012-05-12 10:51:41

Kedves Zilberbach!

Valóban, a specreles időlassulásra szokás példaként emlegetni.

Nos, a fizikai jelenség létező. Csak hogy a Föld gravitációs mezejében v=0,999c és v=0,9999c közötti sebességű részecskékkel.

Amik tetejében h=1e4 m magasságról érkeznek a felszínre

azaz a gravitációs időlassulás t=t0/(1-g*h/c2) értékét mint álló test érzékelné a részecske.

csakhogy rel.Dopplerrel "sűrűbb" számára a gravitációs mezőt alkotó sugárzás.

faktor=((c+v)/(c-v))(1/2) Ez, v=0,9999c esetében faktor=((1,9999)/(0,0001))(1/2)=141

vagyis t=141*2e-6/(1-10*1e4/9e16)=0,000282 [s]

s=3e8*0,000282=84'600 [m]

Tehát a gravitációs időlassulás és a rel.Doppler együttes hatása nem csak a feltételezett 10 km-ről leérkezést, hanem 84,6 km magasról leérkezéshez elegendő időlassulás eredményez.

Vagyis nincs szükség az ikerparadoxon létezésére.

Sőt! Ha e mellé a gravitációs időlassulás mellé még az ikerparadoxon is létezőként hatna, akkor a müonok sokkal kisebb sebességgel is elérhetnék a felszínt.

Amikor ilyeneket olvasol mindig jusson eszedbe, hogy Einstein életében nem volt, nem létezett űrhajó. Minden feltételezése (posztulátum = bizonyítás, igazolás nélkül elfogadandó feltételezés!) csupán egy olyan világban született ember feltételezése volt, amely világban mobil telefon, számítógép, TV, lézer, űrhajó nem volt. A repülés is léghajó szinten "működött" éa világszenzációnak számított 1909-ben a La manche azaz a csatorna átrepülése is.

Vonatokon utazgatva szemlélte a relatív mozgásokat.

Fel sem merülhetett benne, hogy valaha majd például én a számlálós módszerrel bemutatom az ikerparadoxon lehetetlenségét.

Előzmény: [856] Zilberbach, 2012-05-12 09:23:18
[856] Zilberbach2012-05-12 09:23:18

Kedves Gezoo!

Hogyan magyarázható az iker-paradoxon egyik régen sokat emlegetett és sokszor kimért bizonyítéka:

A föld légkörének fölső rétegében 2 mikroszekundum - azaz nagyon rövid átlagos élettartamú - µ-mezonok keletkeznek, a kozmikus sugárzás hatására. 2 mikro-szekundum alatt még fénysebességgel is csak 650 méter utat tudnak megtenni. Mégis lejutnak a földfelszínig, és jól mérhetők. Ezt (régebben) csak azzal tudták magyarázni, hogy ezekben a nagyon gyors mozgású µ-mezonokban lelassul az idő múlása, és ezért nem bomlanak le a kb. 20-30 km-es úton - tehát az iker-paradoxon létezik és működik. Mi akkor erre a jelenségre a magyarázat, ha nem az iker-paradoxon? Ráadásul ezek a részecskék gyorsan közelednek a mérő-műszerekhez és nem távolodnak, tehát a 845.sz hozzászólásodban említetteknek megfelelően számunkra inkább gyorsultnak tűnik benne az idő.

Előzmény: [845] Gézoo, 2012-05-10 18:50:20
[855] Gézoo2012-05-12 08:05:12

A javaslom a "tehetetlenség" és a "tehetetlen tömeg" kifejezéseket tartalmi eltérésük miatt különböztessük meg.

A tehetetlenség a tömeg nagyságához köthető fogalom. A lendület mértékét a tömeg és a relatív mozgásának sebessége képezi. Ezért a nagysága függ a tömeg nagyságától és attól a relatív sebességtől ami a kölcsönható és a tömeg között van:

I=m*v alakban.

"Rokona a mozgási energiának, amely szintén a tömeg nagyságától és a kölcsönhatónak a relatív sebességtől függ:

E=(v/2)*I = (v/2)*m*v = (1/2)*m*v2)

Amely összefüggésben azért nem E=m*v2 hanem csak a fele szerepel, mert a másik fele a kölcsönható oldal energiája.

Azaz legyen két, egyforma tömeg egymás mellett, és az egyikre ülve (nekünk legyen nulla a tömegünk a példa átláthatósága kedvéért,) elrúgjuk a másik tömeget.

Ezt egy külső szemlélő úgy látja, hogy az elrúgás helyétől mindkét tömeg azonos sebességgel és ezzel azonos lendülettel (azaz impulzussal) és ezzel azonos energiával távolodik.

Vagyis a befektetett energia egyik felét az egyik, másik felét a másik tömeg vitte magával. Ezért felezzük meg a mozgási energiát amikor csak az egyik tömeget látjuk mert tudjuk, hogy annak a félnek a sebessége ill. az energiája csak fele a mozgását létrehozó teljes energiának.

Ezekből következően a lendület megváltoztatásához a tehetetlenség ellenében a tömeg nagyságával és a sebesség változtatás felével egyenesen arányos mennyiségű energiát kell befektetnünk.

Ami egyben arra is rámutat, hogy a v sebességgel mozgónak a mozgási irányában két oldalon nem azonos a kölcsön ható képessége. Hátulról ugyanis ahhoz, hogy hathassunk rá, "utána kell futnunk", ami közben elölről elég elé állnunk.

(Azaz az utána futás létrehozásának energia igényét negatív energiának tekintve - mert ezt befektettük, azaz a kölcsönhatásból kapott energiából le kell vonni az egyenleg képzésekor - a tehetetlenség "hátulról" értelmezhető határolt negatív energiájúnak is, ahol a határ az utolérés energia igénye.)

Sőt! Még akkor is hat ránk, ha a sebességénél kisebb sebességgel haladunk előtte. Ekkor például a két sebesség különbségének nagysága adja azt a lendület és energia értéket amit a hatásban érzékelünk.

Ilyen értelemben a tömeg nagyságát a tehetetlenségének mértékeként is szokás definiálni.

A tehetetlenség okaként az égre-földre keresett Higgs bozont és a Higgs mezőt teszik felelőssé.

De ez csak egy elv. Aminek az érvényességét eddig nem sikerült igazolni.

Van sok más elv is.

Egyik az a spinfoton sugárzás elvén alapuló amit ebben a topicban korábban leírtam.

Ha visszalapozol a levezetéssel együtt megtalálod. Minimum gondolat ébresztőnek szánva érdemes elolvasnod.

Előzmény: [854] Zilberbach, 2012-05-11 22:27:01
[854] Zilberbach2012-05-11 22:27:01

Mi a magyarázata a tehetetlen tömeg (látszólagos?) növekedésének?

Előzmény: [850] Gézoo, 2012-05-11 17:23:37
[853] Zilberbach2012-05-11 22:11:55

A véleményed tehát az, hogy (egyenletes sebesség esetén) mindkét rendszerből ugyanazt mérik, a másik rendszerre vonatkozóan. Ha jól értem akkor azt, hogy a tehetetlen tömeg az Einstein szerint jósoltakkal egyezően nő, a gravitáló tömeg viszont nem.

Előzmény: [850] Gézoo, 2012-05-11 17:23:37
[852] Zilberbach2012-05-11 22:10:14

A véleményed tehát az, hogy (egyenletes sebesség esetén) mindkét rendszerből ugyanazt mérik, a másik rendszerre vonatkozóan. Ha jól értem akkor azt, hogy a tehetetlen tömeg az Einstein szerint jósoltakkal egyezően nő, a gravitáló tömeg viszont nem.

Előzmény: [850] Gézoo, 2012-05-11 17:23:37
[851] Gézoo2012-05-11 17:38:37

Amikor Einsteinnek és kortársainak a gondolkodását, elméleteit nézzük, ne feledjük el azt a kort sem amiben ők felnőttek és éltek.

Amikor Einstein megszületett, akkor még minden csillagász esküdött arra, hogy a világ egyetlen galaxisból áll. A relativitás elméletének megjelenése után 4 évvel repülté át először a La manche-t azaz a csatornát. És az első űrhajó Einstein halála után öt évvel vitte fel Gagarint..

Nyilván nem gondolkodhatott olyan lehetőségek létezésében amiket azóta valósított meg az emberiség. A kisfiam amikor Napóleonról tanult és a csatái kerültek szóba azt kérdezte, hogy miért nem hívta fel telefonon Wellington hercegét. Annyira ebben a korban élve, fel sem merült benne, hogy akkor még egy üzenet továbbítása lóval napokba vagy esetenként hónapokba tellett.

Előzmény: [850] Gézoo, 2012-05-11 17:23:37
[850] Gézoo2012-05-11 17:23:37

Nagyon jó felvetés!

A tömeg tehetetlenségét kölcsönhatás hiányában számolással, a relatív energiájának a számításával végezhetjük el. Ez az ütköztetéses próbákkal igazoltan jó számítás Einstein 1905-ös specreljéből másolva:

Viszont az M tömeg gravitáló hatását így nem tudjuk ellenőrizni. Ezért az "elsuhanáskor" a melléhelyezett órák lassulásából t=t0/(1-GM/h'/c2)/sin(arccos(v/c)) függvénnyel megmérhetnénk.

Igen ám, de idő lassulást nem tudott eddig senki sem kimutatni állandó (egyenletes) sebesség esetében.

Azaz ezt azt jelenti, hogy az egyenletes sebességű rendszerekben nyugvó testek tömege nem változik. Viszont a mindenkori energiájukat a mozgási és a tömegben tárolt ( E=m*c2 ) energiáik összege szerint, azaz a mozgási energiák növekedésével az idézett függvény szerint változik.

Ismét meg kell említenem, hogy az egyenletes sebességű mozgásnál mindkét oldal szimmetrikusan ugyanazt mérheti. (Azaz az egyik ugyanazt mint a másik oldal.)

A kérdésed lényegét tekintve, biztosan láttál már autómérleget. (A tüzépekben a teherautók megmérésére szolgáló "túlméretes mázsa".) Ha ezen megy neki az odakészített betonfalnak egy autó, olyan esetben amikor csak 1km/h sebességgel haladt és olyan esetben is amikor 100 km/h sebességgel haladt, akkor a mérleg nem fog különböző tömeget mérni. Pedig az első esetben csak a lökhárító deformálódik, a második esetben ripityára törik az autó.

Azaz nem a gravitáló azaz súlyos tömege, hanem csak a lendülete volt különböző a két esetben.

Előzmény: [849] Zilberbach, 2012-05-11 14:09:11
[849] Zilberbach2012-05-11 14:09:11

Adott egy S és egy M jelű rendszer. Mindkét rendszerben ugyanazok az eszközök találhatók: egy 1 kilogramm tömegű etalon test, és a pontos (táv)mérésére szolgáló műszerek. Kezdetben a két rendszer békésen pihen egymás mellett, és a rendszerekben tartózkodó fizikusok megelégedve tapasztalják, hogy mind a saját, mind a szomszédos rendszerekben lévő etalon testek tömege pontosan 1 kilogramm. Ezután az M rendszer gyors mozgásba kezd, és relativisztikus sebességet ér el (legyen mondjuk: 0,8 c). Ezzel a nagy sebességgel húz el a nyugalomban maradt S rendszer mellett - olyan közelségben, hogy mindketten újra megmérhessék a szomszédos rendszer etalon-testeinek tömegét. Mi lenne a mérések kimenetele?

1.) Mindkét rendszerből 1 kg-nál nagyobb tömegűnek mérnék a másik rendszer etalon-testét?

2.) A nyugalomban maradt S rendszerből 1 kg-nál nagyobb tömegűnek mérnék a 0,8 c sebességgel mozgó M rendszer etalonját, viszont M rendszerből 1 kg-nál kisebb tömegűnek mérnék az S rendszer etalonját?

3.) Valami más kimenetele lenne a a méréseknek.

A sejtésem az, hogy az 1.) a jó válasz. Ha így van, akkor mi az oka ennek - a valójában csak a mérés tökéletlen kivitelezhetőségéből adódó - látszat tömegnövekedésnek?

[848] Gézoo2012-05-10 20:07:17

Szívesen! Kérdezz bátran! Örömmel segítünk a megértésében!

Előzmény: [847] Zilberbach, 2012-05-10 19:57:02
[847] Zilberbach2012-05-10 19:57:02

Köszönöm, egyelőre nincs kérdésem.

Előzmény: [846] Gézoo, 2012-05-10 19:35:41
[846] Gézoo2012-05-10 19:35:41

:) Hát igen. Nagy kérdés, hogy milyen tudásszintűknek szól a könyv. Az sem hátrány, ha az előadó alaposan ismeri nem csak az adott tananyagot, hanem azt is, hogy a benne állókhoz milyen út vezetett. Mindent megértettél? Van még kérdésed?

Előzmény: [844] Zilberbach, 2012-05-10 18:49:51
[845] Gézoo2012-05-10 18:50:20

Az ikerparadoxonban csak addig látszik fiatalabbnak a másik amíg fennáll a relatív sebesség. Ugyanis az öregedésről tudósító fény késve érkezik a \varphi szögű terjedés következtében.

Az a magyarázatok külön hibája, hogy a Földön maradt testvér öregszik jobban, mert a fizikai valóságban pont fordított a helyzet ha a távozó testvér rövid ideig ható gyorsulással éri el az utazási sebességeit.

Ugyanis a Föld gravitációjában lassabban múlik a idő mint az űrben az űrhajó parányi (1e-24 -szeres ) gravitációjában.

Tetejében mindkét testvér fiatalabbnak látja a másikat a távolodás idején mint saját magát.

Ezt úgy lehet a legkönnyebben megérteni, ha mindkét testvér órájának lépését (időalap léptetését) egy-egy villanás kíséri és ezeket a villanásokat a másik tesó is érzékeli.

Azt biztosan tudod, hogy a rel. Doppler torzítja el a beérkező villanások ütemét, de a villanások darabszámát se a Doppler, se a relatív mozgás nem tudja megváltoztatni.

Azaz a távolodáskor például v=0,8c sebesség esetében az órajelek frekvenciája

f=f0*gyök((c-v)/(c+v)) sebességre lassulnak látják egymást: azaz f=f0/3 a v=0,8c esetében.

Vagyis amíg távolodnak egymástól, mindkettő a másik óráját és minden folyamatát 1/3 járási sebességre lassultnak látja. Ezzel az öregedését is.

Visszafelé úton pedig mindkettő a másik óráját f=f0*gyök((c+v)/(c-v))=3*f0 azaz háromszor gyorsabban ketyegőnek látja mint a saját óráját.

Miután minden villanást mindkét óránál megszámolunk, így egymás mellé érve mindkét óra ugyanazon villanásszámot mutathatja, semmi mást.

:) Nem veszhetnek el villanások egyiküknél sem. Azaz nem vesznek el a lassulás és a gyorsulás ellenére sem másodpercek egyikük idejéből sem.

Összefoglalva:

Az ikerparadoxon egy geometria, rossz és nagyon hibás alkalmazásával kapott hibás számítási eredmény.

Előzmény: [840] Zilberbach, 2012-05-10 17:36:36
[844] Zilberbach2012-05-10 18:49:51

Akkor az ikerparadoxon olyan formában ahogy leírtam, nem létezik? Ehhez képest sok fizika-könyv emlegette. Persze ők is tévedhettek.

Előzmény: [843] Gézoo, 2012-05-10 18:13:54
[843] Gézoo2012-05-10 18:13:54

Az idő..

Nos, az idő meg se moccan. Sőt! Einstein az idő helyett az órákat emlegeti. Azaz azt, hogy mit mutatnak az órák.

Ha láttad az előző ábrát, elolvastad azt is, hogy a mozgás irányára merőleges ordináta \varphi=arccos(v/c) szöggel hátra hajlott, akkor nyilván beláthatod azt is, hogy az óra számlapjáról a megfigyelőhöz a fény nem akkor érkezik amikor a mutató lép, hanem a hátra hajlás szögével később. Ezért ha megszűnik a köztük lévő sebesség akkor azt látjuk, hogy hihetetlen sebességgel "előrepörög" a mutató a megfigyelőnél végig nyugalomban lévő óra mutatója mellé.

Persze csak Minkowski geometriájában, mert a valóságban a sebesség változás azaz a gyorsulás hatására valóban lelassul minden folyamat, így az óra járása is (1-gh/c2) szorzóval gravitációs gyorsulás esetében és az ezzel egyenértékű mértékben az erő okozta gyorsulás hatására szintén.

Összefoglalva: Az idő nem lassul le, az állandó sebességű relatív mozgás esetében, csak lassultnak látszik, és ezzel lassultnak mérhető.

Előzmény: [841] Zilberbach, 2012-05-10 17:38:51
[842] Gézoo2012-05-10 17:55:08

Kedves Zilberbach!

Alma jól válaszolt. Mindegy, hogy a gyorsuláson átment, vagy a gyorsulást okozó erőhatás nélkül hagyott rendszerből mérjük a másik rendszerbeli rúd hosszát, ugyanazt az eredményt kapjuk: a saját rúd hosszának sin(arccos(v/c))-szeresének mérhetjük a másik rendszerben nyugvó rúd hosszát.

Ennek oka a mozgás sebességének és a méréshez használt fény sebességének a viszonya.

Minél közelebb van egymáshoz a két sebesség annál inkább hátra hajlottnak látszik a mért rúdról kilépő fény iránya.

Ezzel olyan, mintha a derékszög hátrafelé arc cos(v/c) szöggel hajolna, éppen úgy, mint a képen látható hullámok.

Hogy miért nem a Lorentz féle valós kontrakció lép fel?

Nos, egyszerű! Vegyél kezedbe egy gumira rögzített radírt, vagy más terhet. A gyorsulás alatt a gumi megnyúlik, de a gyorsulást okozó erő megszűnésével visszaáll az eredeti hosszra.

(Mondjuk a gumi pont rossz példa, acélrugó sokkal jobb lenne, mert a gumiban lévő hosszú láncok az erő hatására mindig "megfolynak" azaz átrendeződnek és ezzel a gumi kicsi megnyújtott marad.)

Azaz ha a gyorsulást okozó erőhatás alatt álló méterrúd mellé tennél egy a gyorsulás irányára merőlegesen világító lézer pointert, akkor a gyorsulás kezdetén a fénypontja "hátra maradna" mintha lehajlana a fényútja. De a gyorsulás megszűnésével az eredeti, "nyugalmi állapotába visszaállna a fénypont"

Érdekesség, hogy interferométerben éppen ezt a lehajlást és a megszűnését az interferencia csíkok elmozdulásával ki tudjuk mutatni már pár cm/s2 gyorsulás esetében is.

Az Alma által említett Minkowski féle geometriával ez a "hátra hajlás" szépen kezelhető.

Amikor anno Einstein előállt a relativitás elméletével 1905-ben, az akkori csillagászok egy része úgy tekintett az elméletre, mint a már akkor több mint 200 éve ismert fényaberráció egyik leírási változatára. Nem véletlen, hogy amikor még csak készült a kézirat, ezt már többen jelezték Einsteinnek és így már a 7.§-ban szerepeltette a Doppler és a fény aberráció "relatív" változatát. Miközben az egész dolgozatának a lényegét, az apropóját jelentő elektrodinamikáról csak a 10.§-ban írt. Ezzel is jelezve, hogy mennyire nagy nyomást jelentett az akkori neve csillagászok véleménye.

Egyébként nem "Einstein jósolta", hanem Lorentz írta le, csak Einstein beépítette. Éppen úgy mint ahogyan az E=m*c2 sem Einstein "találmánya", (bár majdnem mindenki, még a szakkönyvek közül is több úgy írja, ) hanem Lebegyev-nek a műve még 1902-ből. Ezt is csak beépítette Einstein.

Tehát összefoglalva:

A mérőjel (a fény) c sebességének és a mozgást végző test (ill. rendszer) sebességének a gyök(1-(v/c)2) (vagy ahogy még felírható sin(arccos(v/c) praktikusan,) viszonyától függő mértékben látszólag hátragörbült a koordináta rendszer mozgásra merőleges iránya.

Azaz nem fizikai változást, hanem egy minden mérésben benne lévő, a jelenlegi méréstechnikával kikerülhetetlen mérési hiba.

Viszont ez a mérési hiba Feynman fényórájával leírva egy olyan derékszögű sebesség-háromszöget alkot, ami mind addig garantálja ennek a hibának a létezését amíg a mérést végző és a mérés tárgya egymáshoz képest mozog.

Megemlítem, hogy nagyon kicsi sebességeknél is érvényesül ez a hatás, csupán olyan kicsiny eltérést okoz amit egyszerűbb esetekben nem veszünk észre.

Előzmény: [838] Zilberbach, 2012-05-10 16:41:42
[841] Zilberbach2012-05-10 17:38:51

Javítás: "(Az egyenértékűség nem is olyan nyilvánvaló, ugyanis az idő lassulása szempontjából a két rendszer nem egyenértékű: az idő lassulása ugyanis csak a gyorsított M rendszerben következik be, és csak az A rendszerből nézve, az M rendszerből nézve viszont az A rendszerben fölgyorsul az idő folyása.)

B.) az M rendszerben gyorsítása miatt valóban történt hosszkontrakció, de emellett még olyan mérési zavarok is kialakulnak, amik azt okozzák, hogy M-ből is - pont ugyanolyan mértékben - rövidebbnek mérik A-rendszer méterrúdját, és a mérési zavarok olyan jellegűek, hogy a megrövidült mozgó méterrúdjukkal is rövidültnek mérik az M rendszer méter-rúdját. Hát ezt nehéz józan ésszel elképzelni, hogy tényleg így legyen, de elvileg ez a lehetőség is fönnáll, ekkor a két rendszer több szempontból sem egyenértékű, nem szimmetrikus a mérési torzulásokra."

Helyesen: (Az egyenértékűség nem is olyan nyilvánvaló, ugyanis az idő lassulása szempontjából a két rendszer nem egyenértékű: az idő lassulása ugyanis csak a gyorsított M rendszerben következik be, és csak az S rendszerből nézve, az M rendszerből nézve viszont az S rendszerben fölgyorsul az idő folyása.)

B.) az M rendszerben gyorsítása miatt valóban történt hosszkontrakció, de emellett még olyan mérési zavarok is kialakulnak, amik azt okozzák, hogy M-ből is - pont ugyanolyan mértékben - rövidebbnek mérik S-rendszer méterrúdját, és a mérési zavarok olyan jellegűek, hogy a megrövidült mozgó méterrúdjukkal is rövidültnek mérik az M rendszer méter-rúdját. Hát ezt nehéz józan ésszel elképzelni, hogy tényleg így legyen, de elvileg ez a lehetőség is fönnáll, ekkor a két rendszer több szempontból sem egyenértékű, nem szimmetrikus a mérési torzulásokra.

Előzmény: [838] Zilberbach, 2012-05-10 16:41:42
[840] Zilberbach2012-05-10 17:36:36

Én másképpen tudom/hallottam, legalábbis az időre vonatkozóan.

Ikerparadoxon: az ikerpár gyorsan utazó tagja fiatal marad, míg az itt maradt megöregszik. Nyilvánvaló hogy nem szimmetrikus a viszonyuk az időben.

Előzmény: [839] Alma, 2012-05-10 17:11:29
[839] Alma2012-05-10 17:11:29

Erre könnyű válaszolni, Gézoonak viszont csak később tudok a vizsgám miatt.

Természetesen mindkét koordinátarendszerből nézve a másik rendszer mérőrúdja megrövidül. Ugyanez érvényes az időkre is, ugyanolyan módon változnak. Teljesen szimmetrikus a két rendszer. Mondok egy analógiát.

Vegyél két közös origóval rendelkező euklideszi koordináta-rendszert, melyek egymáshoz képest el vannak forgatva! Mindkét rendszer x tengelyén vegyél fel egy L hosszúságú szakaszt. Ha a két szakaszt levetíted a másik koordináta-rendszer x tengelyére, mindkét esetben rövidülést tapasztalsz.

Na, specrelben ugyanez van, csak nem euklideszi, hanem minkowski térben. Az effektus lényege ugyanaz.

Előzmény: [838] Zilberbach, 2012-05-10 16:41:42
[838] Zilberbach2012-05-10 16:41:42

Kezdek kissé összezavarodni.

Szeretnék szóbeli kérdést föltenni Almának és Gézoonak is:

Alább fölvázolok egy viszonylag egyszerű gondolatkísérletet - és annak néhány lehetséges kimenetelét, illetve az abból levonható következtetéseket. Kérem hogy válaszoljatok szerintetek melyik kimenetel, illetve következtetés az "igaz"?

Adott egy S és egy M jelű rendszer. Mindkét rendszerben ugyanazok az eszközök találhatók: egy nagyon pontos mérőrúd, úgy kialakítva, hogy a szomszédos rendszerből is jól és pontosan mérhető legyen (ezt nem is olyan egyszerű megoldani, ábra is tartozna hozzá de sajnos nem tudok ábrát rajzolni) és a szomszédos rendszer mérőrúdjainak hosszúságmérésére alkalmas eszközök. Kezdetben a két rendszer békésen pihen egymás mellett, és a rendszerekben tarózkodó fizikusok megelégedve tapasztalják, hogy mind a saját, mind a szomszédos rendszerekben lévő mérőrudak hossza pontosan 1 méter. Ezután az M rendszer gyors mozgásba kezd, és relativisztikus sebességet ér el (legyen mondjuk: 0,8 c). Ezzel a nagy sebességgel húz el a nyugalomban maradt S rendszer mellett - olyan közelségben, hogy mindketten újra megmérhessék a szomszédos rendszer mérőrúdjának hosszát.

A (fontosabb) lehetséges kimenetelek:

1.) Mindkét rendszerben az Einstein képletei által jósolt rövidüléseket mérik a másik rendszer mérőrúdjain.

Lehetséges logikai következtetés(ek):

A.) a rövdüléseket a sebességkülönbség okozta mérési zavar okozza, és nem a Lorentz-kontrakció - ami ezek szerint valójában nincs is. Az 1. lehetőség bekövetkezéseskor ez a valószínűleg helyes magyarázat. Mindkét rendszer egyenértékű a hosszúság kontrakció szempontjából (is?). (Az egyenértékűség nem is olyan nyilvánvaló, ugyanis az idő lassulása szempontjából a két rendszer nem egyenértékű: az idő lassulása ugyanis csak a gyorsított M rendszerben következik be, és csak az A rendszerből nézve, az M rendszerből nézve viszont az A rendszerben fölgyorsul az idő folyása.)

B.) az M rendszerben gyorsítása miatt valóban történt hosszkontrakció, de emellett még olyan mérési zavarok is kialakulnak, amik azt okozzák, hogy M-ből is - pont ugyanolyan mértékben - rövidebbnek mérik A-rendszer méterrúdját, és a mérési zavarok olyan jellegűek, hogy a megrövidült mozgó méterrúdjukkal is rövidültnek mérik az M rendszer méter-rúdját. Hát ezt nehéz józan ésszel elképzelni, hogy tényleg így legyen, de elvileg ez a lehetőség is fönnáll, ekkor a két rendszer több szempontból sem egyenértékű, nem szimmetrikus a mérési torzulásokra.

2.) Mindkét rendszerben pontosan 1m hosszúnak mérik a másik rendszer méter-rúdjait (is).

Lehetséges logikai következtetés(ek):

A.) nincs hosszkontrakció, nincsenek sebbesség okozta mérési zavarok. Ez igencsak valószínűtlen és erősen ellentmondana Einstein megállapításainak.

B.) az M rendszerben gyorsítása miatt történt hosszkontrakció, de emellett még olyan mérési zavarok is kialakulnak, amik ezt pontosan kompenzálják - nem valószínű, hogy így lenne.

3.) S-rendszerből rövidültnek mérik M-rendszer méter-rúdját viszont M-rendszerből hosszabbnak mérik S-rendszer méter-rúdját.

Lehetséges logikai következtetés: A Lorentz-kontrakció valóban, "fizikailag" lezajlik, mérés okozta torzulások viszont nincsenek.

Tovább is van (lenne) mondjam még? Nem ragozom tovább. A válaszokat előre is köszönöm.

[837] Gézoo2012-05-10 05:45:06

Kedves Alma!

"Kíváncsi lennék, hogy az s/t mennyiséget hogy értelmeznéd egy olyan mozgó koordinátarendszerben, melyben a járda és az eredetileg mozgást végző test is mozgást végez. "

Nagyon érdekes felvetés!

Áll a bakter a sín mellett és nézi, ahogy az elhaladó vonaton sétáló ember ruháján mászik egy szentjánosbogár.

És arra kíváncsi, hogy a vonat rendszeréből, hogyan látszik a szentjánosbogár.

Ami arra világít rá, hogy hogyan jelöljük a specrelben az egyes inercia rendszereket és a hozzájuk tartozó mérési értékeket.

Szuper! Nyilván tanultad, hogy a szentjánosbogárnak saját sebessége van abban a rendszerben ahol a bakter nyugszik. Saját sebessége van a vonat rendszerében és az emberhez rögzített rendszerben is.

Tehát adva van négy rendszer ebből három rendszer mozog a bakterhez képest.

A négy rendszer mindegyikéből a másik három rendszer 6-6 azaz összesen 24-féle képpen sorba rendezhető.

Azaz négy rendszer lehet aposztróf nélküli jelöléssel, és mindegyikhez 6-6 variációban ', '', és ''' aposztróf rendelhető a' szerint, hogy melyikből nézzük a többieket, illetve melyik a következő a láncban.

Így, a láncba fűzött rendszereknek az egymáshoz viszonyított sebességei attól függenek, hogy a 24 variáció melyikét alkalmazzuk. (Természetesen még több láncolt rendszer esetében a variációk száma sokszorozódik.)

Ugyanis más lesz a bogár és az ember közötti relatív sebesség a vonat, más a bakter és más a bogár vagy az ember rendszeréből mérve.

Tehát az így alkalmazott aposztróf indexek esetében valóban nem invariáns sem a sebesség, sem a Z, sem a \lambda

Remélem belátod, hogy igazából a kezdeti ellenkezésednek nem az lett volna az alapja, hogy nem azzal a formulával számoljuk a hosszkontrakciót vagy az idődilatációt, hanem az alkalmazott aposztrófos jelölés mást jelent.

És így valóban nem invariáns egyetlen paraméter érték sem.

Ez viszont felveti egy másik problémát.

Ugyanis a mindenkori v sebességet v=c/n alakban megadva, az n állandósága mellett v csak úgy lehet változó értékű, ha c is változó értékű. Ez is és a \gamma=c/c'=1/gyök(1-(v/c)2) függvény is ellent mond a specrel-nek a fénysebesség állandóságáról kimondott posztulátumának.

Másik oldal viszont az, hogy a felsorolt invariancia lehetőségek mindegyike valóban invariáns közvetlen páronként, azaz

K-K1', K-K2', K-K3', .. , K-Kn' esetén.

Előzmény: [836] Alma, 2012-05-09 23:03:54
[836] Alma2012-05-09 23:03:54

Ha jól értem a szép hosszú hozzászólásodban azt írtad le, hogy álló forrás által keltett sugárzás (transzformált) hullámhosszát megmérve mozgó koordinátarendszerben (ismervén a rendszer sebességét is), meghatározható a forrás nyugalmi rendszerbeli frekvenciája (ez egyébként nem is igaz, mert ismerni kell a sugárzás terjedési irányát is, de most nem akarok a 3D-vel kötekedni)

Te az invariáns szót kicsit furcsán használod egyébként. Akkor nevezünk (Lorentz) invariánsnak valamit, ha a (Lorentz) transzformáció invariánsan hagyja. Ehhez semmiféle átdefiniálgatásra nincs szükség, nem kell és egyéb kompenzáló mennyiségeket bevezetni.

Kíváncsi lennék, hogy az s/t mennyiséget hogy értelmeznéd egy olyan mozgó koordinátarendszerben, melyben a járda és az eredetileg mozgást végző test is mozgást végez.

Vagyis, definiáld kérlek az s'' és t'' mennyiségeket egy u sebességgel mozgó rendszerben úgy, hogy s'' / t''= s/t fennálljon!

Előzmény: [835] Gézoo, 2012-05-09 15:40:51
[835] Gézoo2012-05-09 15:40:51

Kedves Alma!

"Testnek neveztem azt, ami a te rendszeredben mozog. "

Ami a járda rendszerében azzal a v sebességgel mozog, amely v sebesség van a két rendszer koordináta rendszereinek origói között, az a test a te rendszeredben nyugvó (álló), a sebessége nulla. Nyilván a járda mint test mozog a te rendszeredben, így a járda v sebességéről és a te rendszered valamely pontján való járda hossz (s'=s*ß) áthaladásának t' idejéről beszélhetünk áthaladási időként, és nem a te rendszeredben nyugvó test áthaladási idejéről, mert a nyugvónak nincs olyan ideje.

"A járdának van sebessége, meg is mondtam, hogy mennyinek gondolom, s'/t', ahogy te is."

Nagyon jó! Így van! Egyetértünk!

******* ******** ******** ******* ********

Ha jól látom, akkor alaposan körbejártuk ezt a témát. És bár, első ránézésre eretnek gondolatnak tűnt számodra a Z=s*t=s'*t' invariánskénti tálalása, a v sebesség "invarianciájának" egy fajta "szinonimájaként" felfogva, csak szokatlan, de éppen úgy invariáns.

A régi görög filozófusok felvetették azt a lehetőséget, hogy amikor kilövünk egy nyilat, akkor a nyíl, sok kis nyilacska sorozataként halad előrefelé.

Ilyen értelmezésben az időegységre eső útszakaszt a sebesség helyett az ismétlődési ütemmel, azaz a frekvenciával is jellemezhetjük.

Vagyis ha a hossz mércéje például egy adott frekvenciájú fényforrásból kilépő fény "f" frekvenciája, akkor a sebesség helyett ezzel is leírható lenne a hossz és az idő viszonya:

s=c°*t' alakban.

Nézzük azt is meg, hogy mit és miért jelöltem ezekkel a jelekkel:

Az f frekvenciájú fényt sugárzó fényforrás nyugodjon K rendszerben. Legyen egy v sebességgel mozgó K' rendszerben a megfigyelő aki így f' frekvenciának méri: f'=f*gyök((v+c)/(v-c)) függvény szerint (rel.Doppler) a fény frekvenciáját - közeledő 1D-s esetben -. Ha ez a K' rendszerben nyugvó megfigyelő képezi a "virtuális" c° sebességet: c°=c*gyök((v+c)/(v-c)) függvény segítségével, akkor az eredeti f frekvencia és Hertz függvényének - c=f*\lambda - felhasználásával az eredeti \lambda hullámhossz: \lambda=c°/f' értéke megegyezik a forrás rendszerében lévő \lambda=c/f hullámhossz értékével.

És miután az f' frekvenciára érvényes, hogy

f'=1/t'

így \lambda=c°*t' alakkal a megmért t' periódusidővel és a "virtuális" c° fénysebességgel közvetlenül megkapjuk a forrás rendszerében mérhető \lambda hullámhosszot.

Ha pedig ezt a \lambda; hullámhosszot a forrás rendszerében távolság, illetve hossz mérésre használtuk, akkor a K' rendszerben mért t' periódus idővel közvetlenül mérhetjük a forrás rendszerében lévő távolságokat.

Például t=1/3e8 [s] esetében a \lambda=1 [m] és v=0,8c rendszerben ezzel c°=c*3 (v=0,8c esetében a gyök((v+c)/(v-c))=3 )

Azaz a megmért t'=1/9e8 [s] periódusidővel a forrás rendszerben lévő hosszok: \lambda=c°*t'=9e8/9e8= 1 [m] azaz a forrás rendszerében mérhető hullámhosszal azonos hossz.

Miután minden v értékkel ugyanezt a \lambda=1 [m] -t kapjuk a forrás rendszeri \lambda=1 [m] hullámhossz esetében, elmondható, hogy a "virtuális" c° fénysebesség használatával a \lambda=c°*t'=v°*t' szintén invariáns.

Végezetül megemlítem, hogy ezek a gondolatok vélhetően éppen úgy szokatlanok a számodra mint az elsőként említett Z=s*t=s'*t' invariáns.

Előzmény: [834] Alma, 2012-05-09 14:52:02
[834] Alma2012-05-09 14:52:02

Testnek neveztem azt, ami a te rendszeredben mozog. A járdának van sebessége, meg is mondtam, hogy mennyinek gondolom, s'/t', ahogy te is.

Előzmény: [833] Gézoo, 2012-05-09 13:14:05
[833] Gézoo2012-05-09 13:14:05

Tehát a járda s' (azaz s'=s*ß rövidült "állapotban" :) ) hosszának egyik vége érkezzen be a Te rendszered egy pontjára t'=0 időpontban és méred az áthaladás idejét t'>0 értéket kapsz. Nyilván az v'=s'/t' a mozgó járda egy pontjának a Te rendszeredben mért sebességét adja..

"De attól, hogy van egy hosszúságadatom, és van egy időadatom, attól a hányadosuk nem lesz a test sebessége a rendszeremben."

Hogy érted, hogy ez a sebesség nem a járda sebessége a Te rendszeredben?

Előzmény: [832] Alma, 2012-05-09 12:55:43
[832] Alma2012-05-09 12:55:43

Nem értek egyet. Mint már többször mondtam, az én rendszeremben nem mozog a vizsgált test, nulla a sebessége és nulla a megtett út.

A te rendszeredben nyugalomban lévő járda hossza az én rendszeremben

s'=s*sin(arccos(v/c)).

Ezzel egyetértek. Ha végigsétálsz a járdán, a mozgás idejét én, az én rendszereben megmérve ha t' értéket kapok, akkor az, a te rendszeredben

t=t'/sin(arccos(v/c)

időtartam lesz. Ezzel is egyetértek. Így definiálva a mennyiségeket még s/t=s'/t'-vel is egyetértek.

De attól, hogy van egy hosszúságadatom, és van egy időadatom, attól a hányadosuk nem lesz a test sebessége a rendszeremben. Abban a trivialitásban megegyezhetünk, hogy a te rendszeredben a test ugyanakkora sebességgel halad, mint az én rendszeremben a járda halad visszafelé.

Előzmény: [831] Gézoo, 2012-05-09 12:42:23
[831] Gézoo2012-05-09 12:42:23

Nagyon jó! Szóval hogy is van ez? a Te rendszered a K' rendszer az s' és t' mérési adatokkal, járda rendszere a K rendszer s és t adatokkal.

A Te rendszeredben mértünk s'=s*ß hosszon áthaladás alatt, t' időt, vagyis v'=s'/t' sebességet.

A járda rendszerében v=s/t

A két v=v' egyenlő nagyságú. (Bár nem szokás v' jelölés, mert az egyenlőség léte alapfelvetés.)

Így egyetértünk?

Előzmény: [828] Alma, 2012-05-09 12:21:07
[830] Gézoo2012-05-09 12:35:34

Jajj bocsának! Elgépeltem! Köszönöm szépen a figyelmeztetést!

Természetesen "invariancia"..

Előzmény: [829] Lóczi Lajos, 2012-05-09 12:22:00
[829] Lóczi Lajos2012-05-09 12:22:00

Szándékosan vagy véletlenül írod a szót így: "invariencia"?

Előzmény: [827] Gézoo, 2012-05-09 10:24:24
[828] Alma2012-05-09 12:21:07

Álljon meg a menet, kezdesz ködösíteni. Világosan fogalmazz! Írd le, hogy szerinted az

s*z=s'*z'

egyenletből hogyan következik az

s/z=s'/z'

egyenlet, mert ez nem puszta matematikai átalakítás.

Megjegyezném, hogy az én, mozgó rendszerem t' idejét projektáltad át a járdához rögzített rendszerbe, vagyis helyesen az idődilatáció:

t=t'/sin(arccos(v/c),

mint ahogy ezt korábban közösen elfogadtuk. Ebből következően az utána levő levezetésed hamis.

Előzmény: [827] Gézoo, 2012-05-09 10:24:24
[827] Gézoo2012-05-09 10:24:24

Jól mondod.. Ez a lényeg.

Az invariencia fennmarad akár s'/t' akár s'*t' a végzett művelet.

Sőt!

s'=s*sin(arccos(v/c)) és t'=t/sin(arccos(v/c) függvények tovább erősítik ezt a megállapítást.

Hogy még szemléletesebb legyen éljünk b=sin(arccos(v/c) alakkal: s'=s*b és t'=t/b

azaz a kérdés, hogy s*t egyenlő-e s'*t' szorzattal?

végezzük el a műveletet s'=s*b és t'=t/b egyenlőségeket felhasználva s*t ?=? s*b*t/b

átrendezve:

s*t ?=? s*t* b/b ahol b/b=1 ott

s*t=s*t eredményt kapunk, tehát ha

s/t=s'/t' invariáns akkor s*t=s'*t' szintén invariáns

Előzmény: [826] nadorp, 2012-05-09 08:59:25
[826] nadorp2012-05-09 08:59:25

Bocs, hogy beleszólok. Előre bocsátom, hogy a relativitáselmélethez nem értek, és ha hülyeséget írnék, már most meggyónom :-). Az nem világos, hogy itt a matematika szabályai "felborulnak"? Gondolok arra, hogy \frac{s}{s'} = \frac{t}{t'} és az állítás szerint a jobb oldal invariáns a reciprok képzésre. De ez nem igaz, mert reciprokot véve a jobb oldal \frac{t^'}{t} lesz. Más analógiát véve, ha veszünk egy háromszöget, aminek két oldala a,b, akkor a hozzá hasonló háromszögek közt az \frac {a}{b} mennyiség invariáns lesz, de ab nem lesz az. Vagy a fentiek nem húzhatók rá a szóban forgó példára?

Előzmény: [825] Gézoo, 2012-05-09 07:08:34
[825] Gézoo2012-05-09 07:08:34

Nagyszerű!

Tehát, ha s*z = s'*z' invariáns, akkor t=1/z alakkal behelyettesítve is invariáns, mivel csak a reciprokáról van szó s*(1/z)= s'*(1/z')

Vagy szerinted van oka annak, hogy a reciprokkal elveszítjük az invarienciát mint tulajdonságot?

Előzmény: [824] Alma, 2012-05-08 21:47:23
[824] Alma2012-05-08 21:47:23

Ennél azért nagyobb lépésekkel is haladhatunk előre, ezekkel egyetértek.

Előzmény: [823] Gézoo, 2012-05-08 21:05:16
[823] Gézoo2012-05-08 21:05:16

Nos, jó. Egyszerűsítsünk!

s/t=s'/t' ebben egyetértettünk, akkor ezzel ebben is:

s * (1/t) =s'* (1/t')

vagy akár z=1/t behelyettesítéssel ebben is:

s*z = s'*z'

Jól gondolom?

Előzmény: [822] Alma, 2012-05-08 15:58:25
[822] Alma2012-05-08 15:58:25

t2*s/t=t'2*s'/t' miért lenne igaz, ha s/t=s'/t'? t\neqt'.

[821] Alma2012-05-08 15:56:06

Belezavarodtam, a t-kbe meg t'-kbe, az s/t = s'/t' ily módon igaz.

Előzmény: [819] Alma, 2012-05-08 14:55:49
[820] Alma2012-05-08 14:56:15

Köszönöm a korrekciót, jogos.

Előzmény: [818] Lóczi Lajos, 2012-05-08 13:42:53
[819] Alma2012-05-08 14:55:49

s/t nem egyenlő s'/t'-vel, nem összeszorozni akartad a kettőt?

Előzmény: [817] Gézoo, 2012-05-08 11:40:30
[818] Lóczi Lajos2012-05-08 13:42:53

A helyes igealak a "kontrahál", nem kontraktál. (A főnév persze a "kontrakció".)

Előzmény: [816] Alma, 2012-05-08 11:26:56
[817] Gézoo2012-05-08 11:40:30

Tehát ha jól értelek, akkor szerinted is érvényes:

v = s/t = s'/t' ?

:D Na akkor szorozzuk meg egy t2 -el minden tagot!

Az egyenlőség nem változhat,

v * t2 = t2*s/t = t'2*s'/t'

Eddig érthető?

Előzmény: [816] Alma, 2012-05-08 11:26:56
[816] Alma2012-05-08 11:26:56

Pontosan, így gondolom. Az én koordináta-rendszeremben, mint megállapodtunk, a tiedben mozgó test áll, 0 a sebessége.

Az én rendszeremben mozog a járdád, így kerül arrébb a test a járdához képest. Nem a testnek van sebessége, hanem a járdának. Ha kettőnk közötti koordináta-rendszert vennénk, akkor még bonyolultabb lenne a helyzet, hiszen akkor a járda és a test is mozogna, valamint a járda nyugalmi hossza is kontraktálódna természetesen.

Előzmény: [815] Gézoo, 2012-05-08 11:20:58
[815] Gézoo2012-05-08 11:20:58

Úgy gondolod, hogy a járda hossza és a hozzá tartozó v sebesség nem interpretálható így?

v = s/t = s'/t' ?

Miért is nem?

Előzmény: [814] Alma, 2012-05-08 10:26:13
[814] Alma2012-05-08 10:26:13

Igen, de itt gyorsan megjegyzem, hogy az s' mennyiség semmiképp nem interpretálható a koordináta-rendszeremben megtett útként, vagy elmozdulásként.

Előzmény: [813] Gézoo, 2012-05-08 07:26:38
[813] Gézoo2012-05-08 07:26:38

Valóban! Elcseréltem. Ügyes vagy, hogy észrevetted!

A Te rendszeredben mért (számított) mérési adatok a vesszős, a járda rendszere a vessző nélküli. Azaz helyesen így kellett volna jelölnöm: t=t'/sin(arccos(v/c)) időszakasz hosszot.

Hiszen így jelöltük a te rendszeredben mérhető járda hosszot is: s'=s*sin(arccos(v/c)) a járda rendszerében mérhető s hosszból.

Tehát van két adatod, amit mértél, számítottál: s' és t'

és van a járda rendszerében két adat s és t.

Eddig egyetértünk?

Előzmény: [812] Alma, 2012-05-07 21:20:39
[812] Alma2012-05-07 21:20:39

Szerintem most véletlen fordítva jelölted "t"-t és "t'"-őt, de amit eddig mondasz, azzal egyet értek. Hogy folytatod?

Előzmény: [811] Gézoo, 2012-05-07 21:02:44
[811] Gézoo2012-05-07 21:02:44

Nagyszerű!

Na most fogom a stopperemet és végigsétálok v sebességgel ezen az s úton, a kapott t időt mutató stopperemet megállítom.

Te azt figyelhetted meg, mert éppen ott voltál a járda elején az induláskor t=0 időpontban, hogy állok a Te rendszeredben és közben megy a stopperem mutatója, mozog alattam a járda. Ebből arra gondolsz, hogy én nyugvó vagyok a Te rendszeredben? Ha így van, akkor jól gondolod. Arra gondolsz, hogy a stopperem a Te rendszeredben nyugodva a Te rendszeredben érvényes időt mutatta? Ha így van, akkor ezt is jól gondolod.

Na most a járdán nyugvó óra vajon mit mutatott amikor a járda végére léptem és ettől megállt?

Jól tippelek ha azt mondom, hogy szerinted a járdán nyugvó óra

ami akkor indult el amikor a járdára léptem és akkor állt meg amikor a járda végére érkeztem az

t'=t/sin(arccos(v/c)) időszakasz hosszot mutat?

Előzmény: [810] Alma, 2012-05-07 19:42:51
[810] Alma2012-05-07 19:42:51

Kedves Gézoo!

Eddig stimmel.

Előzmény: [809] Gézoo, 2012-05-07 19:27:23
[809] Gézoo2012-05-07 19:27:23

Kedves Alma!

Figyelj csak! Másról beszélünk.

Legyen egy s hosszú járda nyugalomban az én rendszeremben, te mozogsz a járdámhoz képest v sebességgel. Ezért a teljes járda mozog a te rendszeredben.

Na akkor a te rendszeredben milyen hosszú az én rendszeremben nyugvó járda? Nyilván elsőre tudtad, hogy te az én járdámat s'=s*sin(arccos(v/c)) hosszúnak méred. Ebben egyetértünk?

Előzmény: [808] Alma, 2012-05-07 18:11:14
[808] Alma2012-05-07 18:11:14

Könyörgöm ne nézz már annyira hülyének, hogy általános iskolás átalakításokat magyarázol el nekem.

s'=s*sin(arccos(v/c))

Ez a rossz képlet, és megint ezt használod. Ez a képlet triviálisan hülyeség. A mozgásnak van kezdete, van vége, és nem ugyanabban az időpillanatban van ez a kettő. NEM HASZNÁLHATOD A LORENTZ-KONTRAKCIÓS KÉPLETET. Az nem erre való. Az egyidejű téridőtávolság transzformálására való. Középiskolában még csak ez fér bele a tananyagba. Nem lehet felépíteni a specrelt, mert nincs rá elég idő. Attól még megvan a tisztességes elmélete, és ha ezt nem ismered, még nem lesz ellentmondásos és rossz az egész.

Az előbb egyeztél bele abba, hogy ha áttérsz a mozgással azonos sebességű rendszerbe, a megtett út 0 lesz.

s'=0

Szerinted az s'=s*sin(arccos(v/c)) képlet ezt adja? Hát nem. Nem ez a jó képlet. Szerintem kezd el olvasni a Taylor-Wheeler Téridőfizika című könyvet, abból meg lehet érteni a specrel alapjait. Függvénytáblázatból nem.

Előzmény: [807] Gézoo, 2012-05-07 17:12:37
[807] Gézoo2012-05-07 17:12:37

Kedves Alma! Sok oka van. Egyik, hogy Z valóban invariáns.

Nézd csak! Z=s*t=s'*t' ezt "állítottam".

s'=s*sin(arccos(v/c)) t'=t/sin(arccos(v/c))

ismeretében helyettesítsünk be Z függvényébe s' és t' helyére:

Z=s*t= s*sin(arccos(v/c))*t/sin(arccos(v/c))

Nem állna fent az egyenlőség?

Nekem úgy tűnik, hogy a Lorentz transzformáció szimmetriája szerint sin(arccos(v/c))/sin(arccos(v/c)) = 1

Azaz a behelyettesített oldalon elvégezve az egyszerűsítést:

Z=s*t= s*t marad..

és miután s*t=s*t ez nyilvánvaló, ezért

Z=s*t=s'*t' függvény helyessége is nyilvánvaló.

Miért írok ilyeneket?

Nos, mert a specrel egy jól megfontolt PR fogással élve úgy lett beharangozva, köztudatba beültetve, hogy

- valós fizikai változást ír le, pedig csupán egy a mozgási sebességek viszonyából fakadó mérési hiba kompenzálására szolgál.

- hú de bonyolult valami, a megértésére egyrészt nincs szükség, másrészt olyan nagyon érthetetlen, hogy ne is próbálja halandó ember megérteni.

Ezek egyike sem igaz. Másrészt a mai hétköznapi gyakorlatban számos olyan eszközt használunk és a mérnöki gyakorlatban pedig tervezünk ami már eddig is használta a specrelt, és egyre több olyan van ami már az áltrelt is alkalmazza.

Ennek ellenére nem csak "mezei" mérnökök, de még a végzett fizikusok is szinte csak hallomásból ismerik az áltrelt.

Azt elismerem, hogy az áltrel egyes részei kicsit elbonyolítottak. Viszont a lényege egyetlen mondattal leírható: Minden folyamatba, jelenségben a méretek képződésétől az energiák hasznosításáig a Idő múlási sebessége az egyetlen alapvetően meghatározó tényező.

Azaz minden amit látunk, érintünk attól éppen olyan, hogy az idő múlási sebessége hogyan változik a környezetében.

Erre pedig, mint minden vita, a mi vitánk is felhívja a figyelmet.

Még az is megeshet, hogy egyesek éppen azért fognak belemélyedni a megismerésébe, mert nem hiszik el azt amit itt olvastak erről a témáról.

Előzmény: [806] Alma, 2012-05-07 14:01:23
[806] Alma2012-05-07 14:01:23

Kedves Gézoo!

Nem értem akkor, miért állítasz olyat, hogy s*t=s'*t', amikor ez láthatóan nem igaz.

Nem értem, miért akarod ebből a relativitáselmélet hibáját kihozni, amikor csak rosszul interpretálod a relativitáselmélet állításait. Relativitáselméletben nincs külön az út megváltozása, és külön az időtartam megváltozása, hanem ezek összefüggnek. Nem tudod simán ugyanannál az esetnél a távolságot kontraktálni és az időt dilatálni. Nincs külön tér és idő, téridő van.

Előzmény: [805] Gézoo, 2012-05-07 11:24:34
[805] Gézoo2012-05-07 11:24:34

Kedves Alma!

Ez így van! Lentebb számítással megerősítettem ezt a megállapításodat.

Előzmény: [804] Alma, 2012-05-07 10:57:51
[804] Alma2012-05-07 10:57:51

Kedves Gézoo!

Egyszerűbben:

Ha van egy egyenes vonaló egyenletes mozgásod, és áttérsz egy olyan koordinátarendszerbe, melynek sebessége megegyezik a mozgás sebességével, abban a koordinátarendszerben a test áll, mozgást nem végez, az általa megtett út 0.

Előzmény: [803] Gézoo, 2012-05-07 00:57:03
[803] Gézoo2012-05-07 00:57:03

Kedves Alma!

A vonaton ülő ember úthossza helyett a vonat hossza lenne a válasz a kérdésemre. Azaz nem kell két mozgást számolni egyetlen mozgás helyett.

Az eredeti felvetésben az szerepelt, hogy van egy s hossz.

Itt nincs idő, csak s hossz. Ez a hossz pedig s'=s*sin(arccos(v/c)) függvénnyel transzformálható.

Az egy másik kérdés volt, hogy ha v sebességgel azaz t=s/v idő alatt végigsétál valaki az s hosszú úton, akkor az út végpontján x=0 elmozdulással transzformált óra állás pedig t'=t/sin(arccos(v/c)) függvénnyel transzformálandó.

Ugyanezt a két függvényt akkor is megkaphatjuk, ha t=0=t' időpontból elindul egy v sebességű mozgás és a relatív időpontokban a K rendszerben t=s/v és K' rendszerben t'=s'/v időpontban a végére megérkezik. Az eredmény így is ugyanaz, mint amit a fenti két függvénnyel kapunk.

Ennek az oka egyszerű K rendszerben a K' rendszerhez tartozó hosszkontrakciót és idő dilatációt mérhetjük. K' rendszerben pedig a K rendszer méretei szenvednek kontrakciót és az óra állásai dilatációt.

Ezért igazából a Z=s*t=s'*t' invariáns egy evidencia, mert a transzformációk szimmetrikusak, ezért a szorzatuk mindig 1 azaz egy értékű szorzótényezőt alkot.

Előzmény: [802] Alma, 2012-05-07 00:07:31
[802] Alma2012-05-07 00:07:31

Kedves Gézoo!

Nem, ebben nincs igazad, a "s'=s*sin(arccos(v/c))" képlet nem adja meg az úthossz transzformáltját.

Épp a minap tartottam egy előadást egy ismeretterjesztő egyetemi konferencián, mely témája súrolta a speciális relativitáselméletet, és ott is igyekeztem illusztrálni két téridőesemény térbeli távolságának változását ha áttérünk más koordináta-rendszerre. Következőképp érveltem:

Vegyünk két téridőeseményt: ülünk egy vonaton, a vonat indulása 14:00 Budapest, érkezés 17:00 Pécs. A Földhöz rögzített koordináta-rendszerben az indulás és a megérkezés térbeli távolsága, mint tudjuk, körülbelül 200 km. Nézzük meg ugyanezt a vonathoz rögzített koordináta-rendszerben! Mit látunk onnan? Az utas ül a székén, nem mozdul egész út során, és valahogy mégis eljut Budapestről Pécsre. Abban a koordináta-rendszerben 0 a sebessége, mozgást nem végez. Nincs megtett út. Ebből pedig ugye következik, hogy a vonathoz rögzített koordináta-rendszerben az indulás és érkezés téridőesemények térbeli távolsága 0.

A "s'=s*sin(arccos(v/c))" képlet természetesen nem ezt adja, láthatod, ez a képlet helytelen.

Előzmény: [800] Gézoo, 2012-05-04 08:41:01
[801] Gézoo2012-05-04 09:39:51

Múltkoriban Lajos bácsi felvetette a Nm mint Joule értelmű energia és a Nm mint nyomaték mértékegységes hasonlóságából adódó félreértés lehetőségét.

A "t" tempus-time-idő jele is éppen ilyen félreértés okozó.

Éppen úgy jelölünk vele időpontot mint időszakaszt.

Alma válasza kapcsán felmerülhet, hogy a t időpontot vagy t időszakaszt jelöl a válaszomban írt függvény számlálójában.

Nyilván s=v*t értelemben használva v sebességgel t időszakasz alatt való elmozdulást jelent.

Ebben az értelemben ha képeznénk x'-értékeinek különbözetét az

x'=(x-v*t)/sin(arccos(v/c)) függvénnyel, akkor időszakaszok különbözeteit képeznénk ezzel a kérdésben felvetett egyetlen t időszakasz használata helyett.

Aki rápillant azt látja itt is "t" ott is "t" azaz ugyanaz.. Pedig elvben sem lehet ugyanaz a két "t" értelmezése.

[800] Gézoo2012-05-04 08:41:01

Kedves Alma! Az s úthossz transzformációja

s'=s*sin(arccos(v/c)) függvénnyel végzendő.

Az x koordináta transzformációja pedig

x'=(x-v*t)/sin(arccos(v/c)) függvénnyel végzendő.

A kettő különbsége látványos.

Előzmény: [799] Alma, 2012-05-04 08:35:28
[799] Alma2012-05-04 08:35:28

Kedves Gézoo! A koordinátaváltozás és a koordináta ugyanúgy transzformálódik.

Előzmény: [797] Gézoo, 2012-05-04 06:30:34
[798] Gézoo2012-05-04 06:33:25

Kicsit félreérted. t#t' azt jelenti, hogy K rendszerben nyugvó megfigyelő óráján

eltelik t idő akkor K' rendszerbeli megfigyelő óráján

t'=t*1/sin(arccos(v/c)) idő telik el.

Ezért t # (ez a jel a nem egyenlő) t'

Előzmény: [796] Zilberbach, 2012-05-03 14:59:39
[797] Gézoo2012-05-04 06:30:34

Kedves Alma! Az "s" az elmozdulás, azaz koordináta változás és nem koordináta.

Előzmény: [795] Alma, 2012-05-03 14:55:27
[796] Zilberbach2012-05-03 14:59:39

Ezt a 791-es hozzászólásodat már átnéztem, már kappisgálom - valamennyire. Az általad írt: t#t' azt jelntené hogy t és t' fizikailag egyenértékűek és "szimmetrikusak" egymásra a transzformációk szempontjából?

Előzmény: [791] Gézoo, 2012-05-03 10:07:53
[795] Alma2012-05-03 14:55:27

Z nem invariáns.

Rosszul értelmezed a hosszkontrakciót és az idődilatációt. Egy téridőtávolság áttranszformálásának tisztességes módja 1+1 dimenziós specrelben a következő:

\ch\left(\chi\right)*ct + \sh\left(\chi\right)*x =ct',

\sh\left(\chi\right)*ct + \ch\left(\chi\right)*x =x',

ahol \th\left(\chi\right)=v/c, \chi a rapiditás.

Előzmény: [791] Gézoo, 2012-05-03 10:07:53
[794] Zilberbach2012-05-03 14:38:08

Kedves Gézoo! Lehet, hogy igazad van, sajnos én ezekkel az általad leírt képletekkel nem vagyok bensőséges viszonyban, egyelőre még gondolkodnom kell az egészen. Például rá kell még jönnöm, hogy mi ez a "Z", stb. Ha valami értelmes gondolatra jutok, illetve ha kérdésem lesz, írni fogok.

Előzmény: [792] Gézoo, 2012-05-03 10:11:55
[793] Gézoo2012-05-03 11:25:15

Vagy, hogy még érthetőbb legyen a relativitás elméletének kikezdhetetlen matematikájában lévő alapvető hiba:

Z= s*t = s'*t' = v*t2 = v'*t'2 = c*t2 = c'*t'2

azaz ahol t2 # *t'2 ott c # c'

Ez annál is nagyobb ciki, mert a relativitás elméletében

az összes transzformációt a:

c/c' = 1/(1-(v/c)2)1/2 függvénnyel végezzük,

amely arányban a posztulált állandó c fénysebesség és

a posztulátummal szemben álló c' fénysebesség aránya szerepel.

Vagyis a teljes specrel a saját posztulátumát megsértő c' sebességgel képzett hányadosra építette minden transzformációját.

Előzmény: [791] Gézoo, 2012-05-03 10:07:53
[792] Gézoo2012-05-03 10:11:55

Természetesen ha hibásnak látod, kérlek jelezd!

Előzmény: [791] Gézoo, 2012-05-03 10:07:53
[791] Gézoo2012-05-03 10:07:53

Szóval mi is ez az invariáns Z?

Legyen két rendszer közötti relatív sebesség v akkor

Ha az s az út hossza és az s út megtételéhez szükséges t idő szorzata Z=s*t akkor a v sebességű IR-ben:

t'=t/(1-(v/c)2)1/2 és s'=s*(1-(v/c)2)1/2

azaz

Z=t/(1-(v/c)2)1/2*s*(1-(v/c)2)1/2

és miután: (1-(v/c)2)1/2/(1-(v/c)2)1/2=1

Z= s*t = s'*t' = v*t2 = v'*t'2

Ez olyan egyszerű evidencia, hogy nyilván belátható.

Na de mit jelent?

K rendszerben v sebességgel t ideig haladó test a K' rendszerben v' sebességgel halad t' ideig.

Ha feltételezzük, hogy K rendszerhez rendelt koordináta rendszerben K' rendszerhez rendelt koordináta rendszer origója v sebességgel mozog t ideig, akkor ugyanezen t idejű mozgás a K' rendszerben t' ideig tart és v' sebességű a fenti evidencia értelmében.

Vagyis ha elfogadjuk azt, hogy t#t' a specrel szerint, akkor a specrelben felhasznált azonosság

amely szerint ha K rendszerben a K' koordináta rendszerének origója v sebességgel mozog t ideig, akkor K' rendszerben is v sebességgel mozog t' ideig a K rendszerhez rendelt koordináta rendszer origója.

Viszont ezen esetben a fent levezetett specreles evidencia nem teljesülhetne.

Vagyis Z # v*t'2 azaz a specrel sebesség fogalmával van egy alapvető számtani és egyben elvi hiba is.

Előzmény: [786] Zilberbach, 2012-04-29 21:40:32
[790] Gézoo2012-05-03 08:00:43

Apropó toporgás.. Egy példa:

Z=s*t=v*t*t

Előzmény: [786] Zilberbach, 2012-04-29 21:40:32
[789] Gézoo2012-05-01 06:46:40

Csak egy kis fizika történelem:

A kerék felfedezésétől Newtonig 8000 év telt el. Newtontól Niels Bohr-ig már csak 300 év. Niels Bohr-tól Einstein általános relativitásáig már csak 3 év telt el. És az áltrel óta stagnál a kvantummechanika.. Na jó Feynman egy picit ragozta a kvantumfizikát, mint anno Maxwell Faraday-Gauss-Lenz-Ampére törvényeit, de sok újat nem tett hozzá. Igazi továbblépés (áttörés) olyan elv lehetne amiben összekapcsolódik a téridő a görbületének okával.

Ha felismerjük, hogy van olyan részecske ami a téridő görbületét okozza akkor ez az összekapcsolódás megtörténik.

Az más kérdés, hogy azt is fel kellene ismerni, hogy a mezőket azaz az erőtereket szintén részecskék árama okozza.

Ezért sem lehet a Higgs mező éter szerűen mindent kitöltő, azaz a modern éter.

[788] Gézoo2012-04-29 23:23:20

Ezt az impulzus frekvencia modulációt (FM), majd a módosított impulzus frekvencia modulációt (MFM) még a mágneses adattárolásban is alkalmaztuk. (Az első floppy ás HDD lemezeken ezeket a modulációs eljárásokat alkalmaztuk.) A soros (RS232 optikai és rádiós hosszabbítóiban szintén impulzus modulációs eljárást alkalmazunk. Ezekben sincs két foton front között semmiféle hullám, semmiféle jel. Egyszerűen kikapcsolt állapotúak a sugárzók. Mégis a vételi oldalon a hangolt körökben hullámzást keltenek az impulzus sorozatok és ezen hullámok szelektív erősítésével érjük el a zavarmentes átvitelt.

Előzmény: [787] Gézoo, 2012-04-29 21:41:55
[787] Gézoo2012-04-29 21:41:55

Pedig egyszerű.. Fogj egy lézer pointert és köss rá egy 1 ns kapuidejű meghajtót.

És bármilyen f frekvenciájú hullámot ki tudsz vele váltani ha t=1/f időnként felvillantod.

Kell egy 500 kHz-es hullámzás? Oké, állítsd a villanások közötti időt t=1/5e5=0,002 msec idejűre.. Kellene egy URH frekvencia mondjuk a sláger rádió frekvenciáján t=1/83e6= 12 ns

Két impulzus között kikapcsolhatod a lézert.. Akkor is létrejön a kívánt frekvenciájú hullámzás.

Évek óta használjuk SSB üzemű "egy impulzus" vagy ahogy mondjuk "egy hullám" modulációban.

Előzmény: [785] Zilberbach, 2012-04-29 21:09:18
[786] Zilberbach2012-04-29 21:40:32

Na már kezdem magamtól is kapisgálni ezt a kapuzott fotonsugárzást: ez a kétréses kísérlet lenne, és valószínűleg attól tükör-kapu, hogy nem egy feketére festett lapon nyílik meg a két rés, hanem egy tükrön. Na így már megjelent a lelki szemeim előtt. Néha idő kell, míg leesik nálam a tantusz.

Előzmény: [785] Zilberbach, 2012-04-29 21:09:18

  [1]    [2]    [3]    [4]    [5]    [6]    [7]